Sie sind auf Seite 1von 128

9

Mathematics
First Quarter

LEARNING ACTIVITY SHEETS


Republic of the Philippines
Department of Education
COPYRIGHT PAGE
MATHEMATICS
Learning Activity Sheets
(Grade 9)
Copyright © 2020
DEPARTMENT OF EDUCATION
Regional Office No. 02 (Cagayan Valley)
Regional Government Center, Carig Sur, Tuguegarao City, 3500

“No copy of this material shall subsist in any work of the Government of the Philippines. However, prior approval of
the government agency or office wherein the work is created shall be necessary for exploitation of such work for
profit.”

This material has been developed for the implementation of K to 12 Curriculum through the Curriculum and Learning
Management Division (CLMD). It can be reproduced for educational purposes and the source must be
acknowledged. Derivatives of the work including creating an edited version, an enhancement of supplementary
work are permitted provided all original works are acknowledged and the copyright is attributed. No work may be
derived from this material for commercial purposes and profit.
Consultants:
Regional Director : ESTELA L. CARIÑO EdD, CESO IV, DepEd R02
Assistant Regional Director : RHODA T. RAZON EdD, CESO V, DepEd R02
Schools Division Superintendent : FLORDELIZA C. GECOBE PhD, CESO VI, SDO Quirino
Asst. Schools Division Superintendent: MARY JULIE A. TRUS PhD, SDO Quirino
Chief Education Supervisor, CLMD : OCTAVIO V. CABASAG PhD, DepEd R02
Chief Education Supervisor, CID : JORGE G. SADDUL, SR.

Development Team
Writers: AGNES M. TALDE, Master Teacher I, Maddela Comprehensive High School-Junior HS
JOVELYN A. DAQUIOAG, Teacher III, Victoria High School- Junior HS
SALIVI G. ELARDE, Teacher III, San Antonio Integrated School - Junior HS
SHERYL JOY P. GALVEZ, Teacher II, Aglipay High School- Junior HS
RIZETTE A. MONTERO, Teacher III, Dumabel Integrated School- Junior HS
KING RICHER N. TABABA, Teacher III, Wasid Integrated School - Junior HS
MAYLEEN V. YANTO, Teacher III, Cabaruan Integrated School, Junior HS
CRIZALYN JOYCE Q. BARTOLOME,Teacher I, Maddela Comprehensive High
School- Junior HS
Content Editor: EDDIE GALUTAN, EdD., Public Schools District Supervisor, SDO Quirino
NERLISA DOMINGCIL, PhD., Principal I, Ifugao Village Integrated School
MAI RANI ZIPAGAN, ENRIQUE GARCIA, JACKILYN ALAMBRA
Language Editor: SHERLY C. CAINGUITAN PhD, Education Program Supervisor - English, SDO Quirino
FE G. BUCCAHAN PhD, Education Program Supervisor -Filipino, SDO Quirino
Focal Persons: EDDIE GALUTAN, EdD., Division Mathematics Coordinator
FELIMENDO M. FELIPE, SEPS-HRD, OIC LR Supervisor, SDO Quirino
ISAGANI R. DURUIN, PhD., Regional Education Program Supervisor- Mathematics
RIZALINO G. CARONAN, Regional Education Program Supervisor- LRMDS

Printed by: Curriculum and Learning Management Division


DepEd, Carig Sur, Tuguegarao City
Address: Regional Government Center, Carig Sur, Tuguegarao City, 3500
Telephone Nos.: (078) 304-3855; (078) 396-9728
Email Address: region2@deped.gov.ph Website: region2.deped.gov.ph
Table of Contents
Page
Compentency
number
1. illustrates quadratic equations ..................... 1
2. solves quadratic equations by:
a) extracting square roots
b) factoring
..................... 7
c) completing the square
d) quadratic formula

3. characterizes the roots of quadratic


..................... 21
equation using discriminant
4. describes the relationship between the
coefficients and the roots of quadratic ..................... 27
equation
5. solves equations transformable to
quadratic equations (including rational ..................... 36
algebraic expressions)
6. solves problems involving quadratic
..................... 41
equations and rational algebraic equations
7. illustrates quadratic inequalities ..................... 48
8. solves quadratic inequalities ..................... 52
9. solves problems involving quadratic
..................... 58
inequalities
10. models real-life situations using quadratic
..................... 63
functions
11. represents quadratic function using;
a) table of values
..................... 69
b) graph
c) equation
12. transforms the quadratic function
defined by ..................... 76
𝑦 = 𝑎𝑥 2 + 𝑘
13. graphs a quadratic function;
a) domain
b) range
c) intercepts ..................... 83
d) axis of symmetry
e) vertex
f) direction of the opening of the parabola
14. analyzes the effects of changing the
values of a, h and k in the equation 𝑦 =
..................... 94
𝑎𝑥 2 + 𝑘 of a quadratic function on its graph

15. determines the equation of a quadratic


..................... 101
function given
a) table of values
b) graph
c) zeros
16. solves problems involving quadratic
..................... 119
function
MATHEMATICS GRADE 9
Name of Learner: ______________________________ Grade Level: _____________
Section: ______________________________________ Date: ___________________

LEARNING ACTIVITY SHEET


Quadratic Equations

Background Information for Learners

A. Illustrating Quadratic Equations in the form 𝒂𝒙𝟐 + 𝒃𝒙 + 𝒄 = 𝟎

A quadratic equation in one variable is a Mathematical sentence in the second degree


power which is written in standard form 𝒂𝒙𝟐 + 𝒃𝒙 + 𝒄 = 𝟎, where a, b, and c are real numbers
and 𝒂 ≠ 𝟎.

Examples of Quadratic Equation


a) 4𝑥 2 − 𝑥 + 5 = 0 → variable x has the highest exponent of 2.

b) 2𝑚(3𝑚 − 1) + 3 = 4𝑚 + 5 → simplify and express in 𝒂𝒙𝟐 + 𝒃𝒙 + 𝒄 = 𝟎


Distributive Property of Multiplication
2
6𝑚 − 2𝑚 + 3 − 4𝑚 − 5 = 4𝑚 − 4𝑚 + 5 − 5 →Addition Property of Equality
6𝑚2 − 6𝑚 − 2 = 0 →reduce the equation by 2
2
3𝑚 − 3𝑚 − 1 = 0 →variable m has the highest exponent of 2

B. Identifying Quadratic Term, Linear Term and Constant in a Quadratic


Equation

In the quadratic equation, 𝒂𝒙𝟐 + 𝒃𝒙 + 𝒄 = 𝟎, 𝒂𝒙𝟐 is the quadratic term, 𝒃𝒙 is the linear
term and c is the constant.
Examples:
1. In the quadratic equation, 𝑥 2 + 2𝑥 + 1 = 0, 𝑥 2 is the quadratic term, 2x is the
linear term, and 1 is the constant.
2. In the quadratic equation, 3𝑥 2 − 2𝑥 − 3 = 0, 3𝑥 2 is the quadratic term, -2x is the
linear term, and -3 is the constant.
3. 2𝑥 (𝑥 + 5) = 10𝑥 − 3 →Simplify
2
2𝑥 + 10𝑥 − 10𝑥 + 3 = 10𝑥 − 10𝑥 − 3 + 3 →Addition Property of Equality
2𝑥 2 + 3 = 0 →Express in 𝒂𝒙𝟐 + 𝒃𝒙 + 𝒄 = 𝟎, if necessary

2𝑥 2 is the quadratic term, 0 is the linear term, 3 is the constant

1
Note: Practice Personal Hygiene protocols at all times.
C. Determining the Coefficients (a, b, c) of Every Term in Quadratic Equations

A quadratic equation in one variable is a mathematical sentence of degree 2 that can be


written in the following standard form:
𝒂𝒙𝟐 + 𝒃𝒙 + 𝒄 = 𝟎, where a, b, and c are real numbers and 𝒂 ≠ 𝟎.
Example: 2𝑥 2 + 5𝑥 − 3 = 0 is a quadratic equation with a = 2, b = 5 and c= -3.

Note: b and c can have zero values.

Learning Competency with Code:


Illustrates quadratic equation (M9AL-Ia-1)

Directions: Read, study carefully and understand the following activities. Answer with
honesty.

ACTIVITY 1: Set Me to Your Standard

Directions/Instructions: Identify which of the following equations are quadratic and which
are NOT. If the equation is quadratic write FACT, if NOT, write BLUFF.

ANSWER
1. 3𝑚 + 8 = 15 __________________________
2. 𝑥 2 − 5𝑥 + 10 = 0 __________________________
3. 12 − 4𝑥 = 0 __________________________
4. 2𝑡 2 − 7𝑡 = 12 __________________________
5. 6 − 2𝑥 + 3𝑥 2 = 0 __________________________
6. 25 − 𝑟 2 = 4𝑟 − 5𝑟 2 + 10 __________________________
7. 3𝑥 (𝑥 − 2) = −7 __________________________
8. 2ℎ (ℎ − 6) = 2ℎ + 3 __________________________
9. (𝑥 + 2)2 = 0 __________________________
10. (𝑤 − 8)(𝑤 + 5) = 14 + 𝑤 2 __________________________

Rubrics for Scoring


(1-5) → 2 points each (10 points)
(6-10) → 3 points each (15 points)
1 point each for incorrect answer

2
Note: Practice Personal Hygiene protocols at all times.
ACTIVITY 2: Work on this!

Directions/ Instructions:
Identify the quadratic term, linear term and constant of each quadratic equation.

Quadratic Linear term Constant


term

1. 𝑥 2 − 𝑥 − 3 = 0

2. 5𝑥 2 − 2𝑥 − 9 = 0

3. 2𝑠 2 − 4𝑠 − 2 = 0

4. 3𝑥 2 + 4𝑥 = −2

5. 𝑥 2 − 𝑥 = 3

6. 2𝑥 2 − 5𝑥 = 𝑥 2 + 2𝑥 − 1

7. 𝑥 (2𝑥 + 3) − 5 = 6𝑥

8. (2𝑚 − 1)(𝑚 + 3) = 3

9. (𝑥 + 1)2 = 1

10. 3𝑥 (2𝑥 + 1) = 3𝑥 − 12

Rubrics for Scoring


(1-5) 1 point each correct answer
(6-10) 2 points each correct answer and 1 point each for incorrect answer

3
Note: Practice Personal Hygiene protocols at all times.
ACTIVITY 3: Find My abc and Set Me to Your Standard!
Directions/ Instructions: Identify the values of a, b and c of each quadratic equation in the form
𝑎𝑥 2 + 𝑏𝑥 + 𝑐 = 0.

For numbers 1-5 A B c

1. 2𝑥 2 − 𝑥 = 0

2. 4𝑥 2 + 5 = 0

3. 𝑥 2 − 5 = 2𝑥

4. 3𝑥 2 − 𝑥 = 1 − 𝑥

5. 5𝑥 2 − 3𝑥 + 4 = 4

For numbers 6-10 Standard Form a b c


𝑎𝑥 2 + 𝑏𝑥 + 𝑐 = 0
6. 2𝑥 (𝑥 − 1) = 4

7. 1 − 3𝑥 2 = 2(𝑥 − 1)

8. (𝑥 − 1)(2𝑥 + 3) = 5𝑥 2

9. (5𝑥 − 1)2 = 3

10. (𝑥 − 1)2 − 𝑥(2𝑥 − 5) = 0

Rubrics for Scoring


(1-5) 1 point each (15 points)
(6-10) 2 points each (20 points)
1 point each incorrect answer

Reflection
Your knowledge on the operations of algebraic expressions is essential in achieving
correct and accurate answers. This reminds you of keeping previous lessons in your own
LIBRAY, your MIND is very important. Tell something about your experience in
accomplishing these activities.
___________________________________________________________________________
___________________________________________________________________________
___________________________________________________________________________
___________________________________________________________________________
___________________________________________________________________________

Reference

4
Note: Practice Personal Hygiene protocols at all times.
Mathematics Learner’s Material 9 First Edition, 2004 Department of Education, pp. 11-18
ANSWER KEY

ACTIVITY 1: Set Me to Your Standard


ANSWER
1. 3𝑚 + 8 = 15 _______BLUFF____________
2. 𝑥 2 − 5𝑥 + 10 = 0 _______FACT_____________
3. 12 − 4𝑥 = 0 _______BLUFF____________
4. 2𝑡 2 − 7𝑡 = 12 _______FACT_____________
5. 6 − 2𝑥 + 3𝑥 2 = 0 _______FACT_____________
6. 25 − 𝑟 2 = 4𝑟 − 5𝑟 2 + 10 _______FACT_____________
7. 3𝑥 (𝑥 − 2) = −7 _______FACT_____________
8. 2ℎ (ℎ − 6) = 2ℎ + 3 _______FACT_____________
9. (𝑥 + 2)2 = 0 _______FACT_____________
10. (𝑤 − 8)(𝑤 + 5) = 14 + 𝑤 2 _______BLUFF____________

ACTIVITY 2: Work on this!


Quadratic Linear Constant
term term
1. 𝑥 2 − 𝑥 − 3 = 0 𝑥2 −𝑥 −3

2. 5𝑥 2 − 2𝑥 − 9 = 0 5𝑥 2 −2𝑥 −9

3. 2𝑠 2 − 4𝑠 − 2 = 0 2𝑠 2 −4𝑠 −2

4. 3𝑥 2 + 4𝑥 = −2 3𝑥 2 4𝑥 2

5. 𝑥 2 − 𝑥 = 3 𝑥2 −𝑥 −3

6. 2𝑥 2 − 5𝑥 = 𝑥 2 + 2𝑥 − 1 𝑥2 −7𝑥 1

7. 𝑥 (2𝑥 + 3) − 5 = 6𝑥 2𝑥 2 −3𝑥 −5

8. (2𝑚 − 1)(𝑚 + 3) = 3 2𝑚2 5𝑚 −6

9. (𝑥 + 1)2 = 1 𝑥2 2𝑥 0

10. 3𝑥 (2𝑥 + 1) = 3𝑥 − 12 𝑥2 0 2

5
Note: Practice Personal Hygiene protocols at all times.
ACTIVITY 3: Find My abc and Set Me to Your Standard!

For numbers 1-5 A B C

1. 2𝑥 2 − 𝑥 = 0 2 -1 0

2. 4𝑥 2 + 5 = 0 4 0 5

3. 𝑥 2 − 5 = 2𝑥 1 -2 -5

4. 3𝑥 2 − 𝑥 = 1 − 𝑥 3 0 -1

5. 5𝑥 2 − 3𝑥 + 4 = 4 5 -3 0

For numbers 6-10 Standard Form A B C


𝑎𝑥 2 + 𝑏𝑥 + 𝑐 = 0
6. 2𝑥 (𝑥 − 1) = 4 𝑥2 − 𝑥 − 2 = 0 1 -1 -2

7. 1 − 3𝑥 2 = 2(𝑥 − 1) 3𝑥 2 + 2𝑥 + 1 = 0 3 2 1

8. (𝑥 − 1)(2𝑥 + 3) = 5𝑥 2 3𝑥 2 − 𝑥 + 3 = 0 3 -1 3

9. (5𝑥 − 1)2 = 3 25𝑥 2 − 10𝑥 − 2 = 0 25 -10 -2

10. (𝑥 − 1)2 − 𝑥(2𝑥 − 5) = 0 𝑥 2 − 3𝑥 − 1 = 0 1 -3 -1

Prepared by:

AGNES M. TALDE
Writer

6
Note: Practice Personal Hygiene protocols at all times.
MATHEMATICS GRADE 9
Name of Learner: ______________________________ Grade Level: _____________
Section: ______________________________________ Date: ___________________

LEARNING ACTIVITY SHEET


Solving Quadratic Equations

Background Information for Learners

A. Solving Quadratic Equations By Extracting The Square Roots

Quadratic equations can be written in the form 𝑥 2 = 𝑘, where k is a real number.


The method of solving the quadratic equation in the form, 𝑥 2 = 𝑘 is called extracting
the square roots.

Note: Quadratic equations are equations with degree 2, therefore there are 2 roots
or solutions.
Example 1. Find the solution of the equation 𝑥 2 − 16 = 0 by extracting the square roots.
𝑥 2 − 16 = 0
𝑥 2 − 16 + 16 = 16 (Addition Property of Equality)
2
𝑥 = 16
√𝑥 2 = √16 (Extract the square root on both sides of the equation)
𝑥 = ±√16 (Since 16 is greater than 0, then the first property can be applied)
𝑥 = ±4
Answer: The equation 𝑥 2 − 16 = 0 has roots/solutions: x= 4 and x= -4.

Example 2: Find the solutions of the equation (𝑥 − 4)2 − 25 = 0


(𝑥 − 4)2 − 25 = 0
(𝑥 − 4)2 − 25 + 25 = 25 (Addition Property of Equality)
2
(𝑥 − 4) = 25 (Write the equation in the form 𝑥 2 = 𝑘)
𝟐
√ 𝑥 − 4 = ±√25
𝑥 − 4 = ±5
𝑥 = 5+4 𝑥 = −5 + 4
𝑥=9 𝑥 = −1
Answer: The equation has solutions: 𝒙 = 𝟗 and 𝒙 = −𝟏.
Example 3: Solve for the roots of (𝑥 − 1)2 = 36
(𝑥 − 1)2 = 36
√(𝑥 − 1)2 = √36 (Extract the square root on both sides of the equation)
𝑥 − 1 = ±6

7
Note: Practice Personal Hygiene protocols at all times.
𝑥 = 1+6 𝑥 = 1−6
𝒙=𝟕 𝒙 = −𝟓
Answer: The equation has solutions: 𝒙 = 𝟕 and 𝒙 = −𝟓.

Example 4: Find the roots of 𝑥 2 = 8


𝑥 2 = 8 → √𝑥 2 = ±√8 (Extract the square root on both sides of the equation)
𝑥 = ±√4 ∙ 2 (Since 8 is not a perfect square number, get the factors
of 8 which is a perfect square number) Factors of 8: 8∙ 1, 4 ∙ 2, where 4 is a perfect
square number.
𝑥 = ±2√2
Answer: The equation has solutions/roots: 𝒙 = 𝟐√𝟐 and 𝒙 = −𝟐√𝟐.

B. Solving Quadratic Equation by Factoring

Some quadratic equations can be solved easily by factoring. To solve quadratic


equations, the following procedures can be followed:
1. Transform the quadratic equation into standard form if necessary,
2. Factor the quadratic expressions,
3. Apply the zero property by setting each factor of the quadratic expression equal to 0.
4. Solve each resulting equation.

Example 1: Common Monomial Factoring


Find the solutions of 𝑥 2 + 9𝑥 = 0 by factoring.
𝑥 2 + 9𝑥 = 0 → 𝑥 (𝑥 + 9) = 0 (Factor out the common monomial factor x)
𝑥=0 𝑥 + 9 = 0 (Equate each factor to 0)
𝒙=𝟎 𝒙 = −𝟗

Example 2: Factoring Quadratic Trinomial


a) where 𝒂 = 𝟏
i. Find the roots of 𝑥 2 − 5𝑥 − 24 = 0 by factoring.
Factors of 24 are: 4∙ 6, 3∙ 8, 12∙ 2, 1∙ 24
Since the sign of the last term is negative, this requires you to get the factors whose
difference is 5 (middle term). Therefore, from the given factors above, 3∙ 8 has difference
of 5.
𝑥 2 − 5𝑥 − 24 = 0
(𝑥 − 8)(𝑥 + 3) = 0 (Write the equation in factored form)
𝑥−8=0 𝑥 + 3 = 0 (Equate each factor to 0 and apply Addition Property of Equality)
𝑥−8+8=0+8 𝑥+3−3=0−3
𝒙=𝟖 𝒙 = −𝟑

Note: The sign of the larger factor copies the sign of the middle term.

ii. Find the roots of 𝑥 2 + 10𝑥 + 24 = 0 by factoring.


Factors of 24: 1∙ 24, 2∙ 12, 3∙ 8, 4∙ 6

8
Note: Practice Personal Hygiene protocols at all times.
Since the sign of the last term is positive, this requires you to get the factors
whose sum is 10 (middle term). Therefore, from the given factors above, 4∙ 6 has
sum of 10.
𝑥 2 + 10𝑥 + 24 = 0
(𝑥 + 4)(𝑥 + 6) = 0 (Write the equation in factored form)
𝑥+4= 0 𝑥 + 6 = 0 (Equate each factor to 0 and apply Addition Property of Equality)
𝒙 = −𝟒 𝒙 = −𝟔

b) where a≠1
Set in the standard form 𝒂𝒙𝟐 + 𝒃𝒙 + 𝒄 = 𝟎
i. Solve for the roots of 2x 2 + 3x − 2 = 0.
Factor the first and 3rd terms. Write them in to adjacent parentheses then get the
product of the extremes and means.
(2𝑥 − 1)(𝑥 + 2) = 0
Product of the Extremes 2𝑥 ∙ 2 = 4𝑥
Product of the Means (𝑥 )(−1) = −𝑥
Since the sign of the last term is negative, get the DIFFERENCE of their
products.

4𝑥 − 𝑥 = +3𝑥 (Middle term)


Equate each factor into zero and solve for x.
2𝑥 1
(2𝑥 − 1) = 0 → =2
2
𝟏
𝒙=𝟐
𝑥+2= 0
𝑥 + 2 − 2 = −2
𝒙 = −𝟐

ii. Find the solutions of 6𝑥 2 − 17𝑥 + 12 = 0 by factoring.


If the last term is positive, then get the SUM of their products.
(3𝑥 − 4)(2𝑥 − 3) = 0
Product of Extremes 3𝑥 ∙ −3 = −9𝑥
Product of Means 2𝑥 ∙ −4 = −17𝑥 (Middle Term)

Equate each factor into zero and solve for x


(3𝑥 − 4) = 0 2𝑥 − 3 = 0

3𝑥 − 4 + 4 = 0+4 2𝑥 − 3 + 3 = 0 + 3
3𝑥 4 2𝑥 3
=3 =2
3 2

𝟒 𝟑
𝒙=𝟑 𝒙=𝟐

C. Solving Quadratic Equation by Completing the Square

9
Note: Practice Personal Hygiene protocols at all times.
This lesson will help you recall your knowledge on the square of a binomial which will
give you a perfect square trinomial.
Review: (𝑎 + 𝑏)2 = 𝑎2 + 2𝑎𝑏 + 𝑏2
Examples: 1. (𝑥 + 1)2 = 𝑥 2 + 2𝑥 + 1
2. (𝑎 − 3)2 = 𝑎2 − 6𝑎 + 9
Suppose in 𝑥 2 − 4𝑥 + ________, how do you determine the last term (c)?
Steps: a) get one-half of the coefficient of x
1
−4 ∙ 2 = −2, and
b) square is (−2)2 = 4, then write on the blank.
Answer: 𝑥 2 − 4𝑥 + 4 →perfect square trinomial
These steps will facilitate you in solving quadratic equations by completing the square
𝑎𝑥 2 + 𝑏𝑥 + 𝑐 = 0. Extracting the square roots and factoring are both useful in completing the
square.

Examples:
Solve for the roots of quadratic equation by COMPLETING THE SQUARE
1. 𝑥 2 − 6𝑥 − 7 = 0 when 𝑎 = 1
Steps: a) add 7 to both sides of the equation and simplify
𝑥 2 − 6𝑥 − 7 + 7 = 0 + 7
𝑥 2 − 6𝑥 = 7
b) get one-half of the numerical coefficient of -6x, square it and add the result
to both sides of the equation.
1
−6 ∙ = (−3)2 = 9
2
𝑥 2 − 6𝑥 + 9 = 7 + 9
𝑥 2 − 6𝑥 + 9 = 16
c) express the left side of the equation as a square of a binomial
𝑥 2 − 6𝑥 + 9 = (𝑥 − 3)(𝑥 − 3)
= (𝑥 − 3)2
d) write the result at the left side of the equation
(𝑥 − 3)2 = 16
e) extract the square root on both sides of the equation
(𝑥 − 3)2 = 16
2
√(𝑥 − 3) = √16
𝑥 − 3 = ±4
f) solve the resulting linear equation
𝑥−3= 4 𝑥 − 3 = −4
𝑥−3+3=4+3 𝑥 − 3 + 3 = −4 + 3
𝑥= 7 𝑥 = −1
Checking:
Substitute the value of x to the equation 𝑥 2 − 6𝑥 − 7 = 0
If 𝑥 = 7 If 𝑥 = −1
2
𝑥 − 6𝑥 − 7 = 0 𝑥 2 − 6𝑥 − 7 = 0
(7)2 − 6(7) − 7 = 0 (−1)2 − 6(−1) − 12 = 0
10
Note: Practice Personal Hygiene protocols at all times.
49 − 42 − 7 = 0 4 + 8 − 12 = 0
49 − 49 = 0 12 − 12 = 0
0 = 0 TRUE 0 = 0 TRUE
2
2. Find the roots of 2𝑥 − 5𝑥 − 3 = 0 by completing the square.
when 𝑎 > 1
a) add 3 on both sides of the equation
2𝑥 2 − 5𝑥 − 3 + 3 = 0 + 3
2𝑥 2 − 5𝑥 = 3
b) divide by 2 on both sides of the equation (coefficient of 𝑥 2 )
2𝑥 2 −5𝑥 3
=2
2
5 3
𝑥2 − 2 𝑥 = 2
5
c) get one-half of the coefficient of − 2 𝑥, square it and add the result to both
sides of the equation
5 1 −5 −5 2 25
−2∙2 = 4
→ ( 4 ) = 16
5 25 3 25 3 25 24 + 25 49
𝑥 2 − 2 𝑥 + 16 = 2 + 16 → + = =
5 25 49 2 16 16 16
𝑥 2 − 2 𝑥 + 16 = 16
5 25
d) express 𝑥 2 − 2 𝑥 + 16 as a square of a trinomial
5 49
(𝑥 − )2 =
4 16
e) then, solve by extracting the square root
5 7
𝑥 −4 = ±4
f) solve the resulting linear equations
5 7 5 7
𝑥−4= 4 𝑥 −4 = −4
5 5 7 5 5 5 7 5
𝑥−4+4 =4+4 𝑥 −4+ 4 = −4+4
12 2
𝑥= 𝑥 = − 4 𝑜𝑟
4
𝟏
𝒙=𝟑 𝒙 = −𝟐

D. Solving Quadratic Equation Using the Quadratic Formula

To solve quadratic equation, 𝑎𝑥 2 + 𝑏𝑥 + 𝑐 = 0 using the quadratic formula, determine


the values of a, b, and c, then substitute these in the equation
−𝑏±√𝑏 2−4𝑎𝑐
𝑥= (Quadratic Formula)
2𝑎

Simplify the result if necessary.


Note: If the given quadratic function is factorable, use factoring. However, if the quadratic
equation is not factorable, it is suggested to use the Quadratic Formula.

Examples: Solve for the roots of the quadratic equations using the Quadratic Formula

1. 2𝑥 2 + 3𝑥 = 27 or 2𝑥 2 + 3𝑥 − 27 = 0, where 𝑎 = 2, 𝑏 = 3, 𝑐 = −27

11
Note: Practice Personal Hygiene protocols at all times.
−𝑏±√𝑏2 −4𝑎𝑐
Use: 𝑥= 2𝑎
− 3 ±√(3)2 −4(2)(27)
( )
then substitute 𝑥 = 2(2)
−3±√9+216
𝑥= 4

−3±√225
𝑥= 4
−3±15
𝑥= 4
−3+15 12
𝑥= = =𝟑
4 4
−3−15 −18 −𝟗
𝑥= = =
4 4 𝟐

2. 3𝑥 2 − 𝑥 − 5 = 0, 𝑎 = 3, 𝑏 = −1, 𝑐 = −5
−𝑏±√𝑏2 −4𝑎𝑐
𝑥= 2𝑎
− −1 ±√(−1)2 −4(3)−5
( )
𝑥= 2(3)
1±√1+12
𝑥= 6
Note: √13 has no perfect square root
𝟏±√𝟏𝟑
𝑥= 𝟔
𝟏+√𝟏𝟑 𝟏−√𝟏𝟑
or 𝑥 = and 𝑥 =
𝟔 𝟔

3. 3𝑥 2 − 4𝑥 + 4 = 0, 𝑎 = 3, 𝑏 = −4, 𝑐 = 4
−(−1)±√(−1)2 −4(3)(4)
𝑥= 2(3)
1±√1−48
𝑥= 6
1±√−47
𝑥= imaginary roots
6

Learning Competency with Code:


Solves quadratic equations by a) extracting the square roots, b) factoring, c)completing
the square, d) using the Quadratic Formula (M9AL-Ia-b-1)

Directions: Read, study carefully and understand the following activities. Answer with
honesty.

12
Note: Practice Personal Hygiene protocols at all times.
ACTIVITY 1: Find my Solutions!
Directions: Solve the following quadratic equations by extracting the square roots.
1. 𝑥 2 = 16 6. 𝑥 2 = 50

2. 𝑟 2 − 100 = 0 7. 𝑡 2 − 12 = 0

3. 𝑠 2 = 64 8. 𝑐 2 − 32 = 0

4. 𝑟 2 = 18 9. (𝑘 + 7)2 = 81

5. 𝑥 2 − 144 = 0 10. (𝑥 − 4)2 = 169

Rubrics for Scoring


2 points each (20 points)
1 point each for answer with minor errors

13
Note: Practice Personal Hygiene protocols at all times.
ACTIVITY 2: Work on this!
Directions/Instructions: Find the solutions of quadratic equations. Use factoring.

A. 𝑎 = 1
1. 𝑥 2 + 7𝑥 = 0

2. 𝑥 2 − 5𝑥 = 0

3. 𝑡 2 + 8𝑡 + 16 = 0

4. ℎ2 + 3ℎ = 30

5. 𝑥 2 − 20 = 8𝑥

B. 𝑎 ≠ 1

6. 2𝑥 2 − 7𝑥 + 5 = 0

14
Note: Practice Personal Hygiene protocols at all times.
7. 3𝑥 2 + 17𝑥 − 6 = 0

8. 10𝑥 2 + 31𝑥 + 15 = 0

9. 4𝑥 2 − 12𝑥 + 9 = 0

10. 3𝑥 2 − 5𝑥 = 0

Rubrics for Scoring


(1-5) 2 points each (10 points) 1 point each for answer with minor error
(6-10) 4 points each (20 points) 2 points each for answer with minor error

ACTIVITY 3: Make Me Perfect!


Directions: Determine a number that must be added to make each a perfect square trinomial.
Show your solutions.

1. 𝑥 2 − 6𝑥 + _______________
2. 𝑡 2 − 24𝑡 + ________________
3. 𝑎2 − 30𝑎 + _______________
4. 𝑥 2 − 12𝑥 + _______________
5. 𝑥 2 + 10𝑥 + _______________
15
Note: Practice Personal Hygiene protocols at all times.
6. 𝑥 2 + 3𝑥 + _______________
7. 𝑥 2 + 14𝑥 + _______________
8. 𝑚2 − 20𝑚 + _______________
9. 𝑟 2 − 5𝑟 + _______________
10. 𝑏2 + 18𝑏 + ______________

Rubrics for Scoring


(1-10) 2 points each 1 point each for answer with minor error
ACTIVITY 4: Am I A Solution? Complete Me!
Directions: Solve for the roots/ solutions of the following quadratic equations by completing
the square. Show your solutions.

1. 𝑥 2 − 2𝑥 − 15 = 0

2. 𝑥 2 − 7𝑥 + 12 = 0

3. 3𝑥 2 + 5𝑥 − 2 = 0

4. 4𝑥 2 − 5𝑥 − 6 = 0

5. 2𝑥 2 − 3𝑥 + 1 = 0

Rubrics for Scoring


(1-5) 4 points each 2 points each for incorrect answer but with solutions

ACTIVITY 5: Does Your Formula Work?


Directions: Find the solutions of each quadratic equation using Quadratic Formula

1. 𝑥 2 + 7𝑥 + 10 = 0

2. 𝑥 2 − 𝑥 − 12 = 0

3. 𝑥 2 − 2𝑥 = 15

16
Note: Practice Personal Hygiene protocols at all times.
4. 𝑥 2 + 20 = 12𝑥

5. 𝑥 2 − 9𝑥 + 14 = 0

6. 6𝑥 2 − 𝑥 − 3 = 0

7. 3𝑥 2 − 4𝑥 = 0

8. 5𝑥 2 − 12 = 0

9. 2𝑥 2 − 𝑥 − 3 = 0

10. 2𝑥 2 + 7𝑥 + 9 = 0

Rubrics for Scoring


(1-5) 2 points each (10 points) 1 point each for incorrect answer
(6-10) 3 points each (15 points) 2 points each for incorrect answer but with
correct substitution

Reflection
After doing the different activities in this lesson, what made you excited in performing
the activities? Tell something about these.
___________________________________________________________________________
___________________________________________________________________________
___________________________________________________________________________
___________________________________________________________________________
___________________________________________________________________________

Reference
Mathematics Learner’s Material 9 First Edition, 2004 Department of Education, pp. 19-64

17
Note: Practice Personal Hygiene protocols at all times.
ANSWER KEY

ACTIVITY 1: Find my Solutions!

1. 𝑥 2 = 16 6. 𝑥 2 = 50

±4 ±5√2

2. 𝑟 2 − 100 = 0 7. 𝑡 2 − 12 = 0

±10 ±2√3

3. 𝑠 2 = 64 8. 𝑐 2 − 32 = 0

±8 ±4√2

4. 𝑟 2 = 18 9. (𝑘 + 7)2 = 81

±3√2 2, 6

5. 𝑥 2 − 144 = 0 10. (𝑥 − 4)2 = 169

17, -9
±12

ACTIVITY 2: Work on this!


A. 𝑎 = 1
1. 𝑥 2 + 7𝑥 = 0

18
Note: Practice Personal Hygiene protocols at all times.
0, −7

2. 𝑥 2 − 5𝑥 = 0

0, 5

3. 𝑡 2 + 8𝑡 + 16 = 0

−4 → one real root


4. ℎ2 + 3ℎ = 30

−10, 3
5. 𝑥 2 − 20 = 8𝑥

10, −2

B. 𝑎 ≠ 1

6. 2𝑥 2 − 7𝑥 + 5 = 0

5
,1
2
2
7. 3𝑥 + 17𝑥 − 6 = 0

1
, −6
3
8. 10𝑥 2 + 31𝑥 + 15 = 0

5 3
−2,−5

9. 4𝑥 2 − 12𝑥 + 9 = 0

3
→one real root
2

10. 3𝑥 2 − 5𝑥 = 0

5
0, 3

ACTIVITY 3: Make Me Perfect!


1. 𝑥 2 − 6𝑥 + 9
2. 𝑡 2 − 24𝑡 + 144
3. 𝑎2 − 30𝑎 + 225
4. 𝑥 2 − 12𝑥 + 36
19
Note: Practice Personal Hygiene protocols at all times.
5. 𝑥 2 + 10𝑥 + 25
𝟗
6. 𝑥 2 + 3𝑥 + 𝟒
7. 𝑥 2 + 14𝑥 + 49
8. 𝑚2 − 20𝑚 + 100
𝟐𝟓
9. 𝑟 2 − 5𝑟 + 𝟒
10. 𝑏2 + 18𝑏 + 81

ACTIVITY 4: Am I A Solution? Complete Me!

1. 𝑥 2 − 2𝑥 − 15 = 0 5, -3
2. 𝑥 2 − 7𝑥 + 12 = 0 3, 4
𝟏
3. 3𝑥 2 + 5𝑥 − 2 = 0 , −𝟐
𝟑
𝟑
4. 4𝑥 2 − 5𝑥 − 6 = 0 −𝟒,𝟐
𝟏
5. 2𝑥 2 − 3𝑥 + 1 = 0 ,𝟏
𝟐

ACTIVITY: Does Your Formula Work?


1. 𝑥 2 + 7𝑥 + 10 = 0
-2, -5
2. 𝑥 2 − 𝑥 − 12 = 0
-3, 4
3. 𝑥 2 − 2𝑥 = 15
-3, 4
4. 𝑥 2 + 20 = 12𝑥
2, 10
5. 𝑥 2 − 9𝑥 + 14 = 0
2, 7
6. 6𝑥 2 − 𝑥 − 3 = 0
1±√73
12
2
7. 3𝑥 − 4𝑥 = 0
4
0, 3
8. 5𝑥 2 − 12 = 0
±2√15
5
9. 2𝑥 2 − 𝑥 − 3 = 0
3
, −1
2
10. 2𝑥 2 + 7𝑥 + 9 = 0

−7±√−23
4

20
Note: Practice Personal Hygiene protocols at all times.
Prepared by:

AGNES M. TALDE
Writer

MATHEMATICS GRADE 9
Name of Learner:_____________________ Grade Level:__________________
Section:_____________________________ Date:_______________________

LEARNING ACTIVITY SHEET


Nature of the Roots of Quadratic Equations Using the Discriminant

Background Information for Learners


A. Determining the Discriminant of a Quadratic Equation
In the previous activities, you have learned the different ways on how to solve quadratic
equations. This lesson will teach you on how to determine the discriminant of a quadratic
equation.
The discriminant of a quadratic equation is the radicand in the quadratic formula which
is b2 – 4ac . Express all equations to ax2 + bx + c = 0 if necessary. To determine the discriminant
of a quadratic equation, take the following steps in the given equations below.
Examples:
Determine the discriminant of the following equations:
1. x2 - 8x + 16 = 0
The given quadratic equation is in the general form ax2 + bx + c = 0.
Then, we have a = 1, b = -8 and c = 16. Substitute the values of a, b, and c in the
expression b2 - 4ac.
b2 - 4ac = (-8)2 - 4(1)(16)
= 64 - 64
= 0
So, the value of the discriminant = 0
00
2. 2x2 + 5x – 5 = 0 0
The given quadratic equation is in the general form ax 2 + bx + c = 0,
where a = 2, b = 5 and c = -5. Substitute the values of a, b, and c in the expression
b2 - 4ac (discriminant).
b2 - 4ac = (5)2 - 4(2)(-5)
= 25 – 4(-10)
21
Note: Practice Personal Hygiene protocols at all times.
= 25 + 40
= 65
So, the value of the discriminant = 65 and this is greater than 0.

3. x2 + 9 = 0
In the equation, the values of a,b,c are 1, 0, and 9 respectively. Since b= 0, therefore
the equation is an incomplete. Use these values to evaluate b2 - 4ac.
b2 - 4ac = 02 – 4( 1) (9)
= 0 – 36
= -36
So, the value of the b2 - 4ac is a negative number and this is less than 0

4. -5x – 12 = -2x2
This is a quadratic equation that is not written in standard form. So, transform the
equation in the form ax2 + bx + c = 0 . The resulting equation is 2x2 .- 5x -12 = 0.
The value of a, b, c are 2, -5 and -12, respectively. Substitute these values to evaluate
b2 - 4ac.
b2 - 4ac = -52 – 4(2) (-12)
= 25 + 96
= 121
So, the value of the discriminant = 121 and this is greater than 0.

B. Determining The Nature of the Roots of A Quadratic Equation

In the previous activities you have learned how to find the discriminant of a quadratic
equation. This lesson will teach you on how to determine the nature of the roots of a quadratic
equation.
Recall that the roots of a quadratic equation in standard form, ax2 + bx + c = 0, can be
−𝑏±√𝑏2−4𝑎𝑐
found using the quadratic formula: 𝑥 = where the radicand b2 – 4ac is called the
2𝑎
discriminant of the quadratic equation. This aids us in determining the nature of the roots of
quadratic equation
The following are to be considered in determining the nature of the roots of quadratic
equation:
If : Then the roots are:
b – 4ac = 0
2
two equal real roots
b – 4ac > 0
2
two unequal real roots
b – 4ac < 0
2
two unequal imaginary roots

Examples: Determine the nature of the roots of the following equations using the discriminant:
1. 5x2 - 10x +5 = 0
This equation is written in standard form. To determine the nature of its roots,
evaluate b2 – 4ac ,
b2 – 4ac = -102 – 4(5)(5)
= 100 – 100

22
Note: Practice Personal Hygiene protocols at all times.
=0
Since the value of b2 – 4ac is zero, we can say that there are two equal real roots
2. x2 - 4x = 0
Substitute the values of a,b,c to evaluate b2 – 4ac.
b2 – 4ac = -42 – 4(1) (0)
= 16 – 0
= 16
The obtained value of b2 – 4ac is 16 and this number is greater than 0 , therefore
there are two unequal real roots.

3. x2 = - 2x – 5
Since the equation is not written in standard form. Transform it first into the form
ax2 + bx + c = 0 . Then, substitute the values of a,b, and c in the expression b2 – 4ac .
x2 = - 2x – 5 To transform this equation, equate it into zero (0)
2
x + 2x + 5 = 0 Substitute the values of a. b, and c in the expression
b2 – 4ac
b2 – 4ac = 22 – 4( 1) ( 5)
= 4 – 20
= -16
The obtained value of the discriminant (b2 – 4ac) is less than 0, the quadratic
equation x2 = - 2x – 5 has no real roots or the roots are two unequal and imaginary roots.

Learning Competency with Code:


Characterizes the roots of a quadratic equation using the discriminant (M9AL-Ic-1)

Directions: Read, study carefully and understand the following activities. Answer with
honesty.

Activity 1: Look then Fill Me Up

Directions: Consider the given equations and fill in the table below. (1 point each for the correct
answers in standard form and value of discriminant. Highest Possible score: 2 x 10 items = 20
points)
Quadratic Equation Quadratic Equation Value of
in Standard Form b2 – 4ac
(ax2 + bx + c = 0 ) (discriminant)
1. x2 + 3x + 12 = 0
2. x2 - 6x +9 = 0
3. 2x2 - 5x -3 = 0
4. 2x2 + 3x -2 = 0
5. x2 - 8x +16 = 0
6.3x2 = -2x -5
7. 3x2 - 12 = 7x
23
Note: Practice Personal Hygiene protocols at all times.
8. 2x2 – 6x = -12
9. 3x2 = -2x - 5
10. 5x2 = 5

Rubric for Activity 1

5 3 2 1
Mathematical - Quadratic equation - Quadratic equation - Quadratic no
Concept Used was transformed was transformed equation was answer
correctly into its correctly into its transformed
standard form and the standard form, but incorrectly into
value of the the value of the its standard form
discriminant is discriminant is but the value of
correct incorrect. the discriminant
is correct.

Activity 2. Discriminant Ko Nature Mo?

Directions: Determine the nature of the roots of the following quadratic equations by solving
its discriminant. Then put a check mark (✓) on the appropriate column.
(1 point each for the correct value of the discriminant and nature of the roots .
Highest Possible Score : 2 x 10 = 20 points)
Nature of the Roots
2
Quadratic Equations ax + bx + c = 0 Value of two two two
b2 – 4ac equal unequal unequal
(discriminant) real roots real roots and
imaginary
roots
Examples:
x2 - 8x + 16 = 0 x2 - 8x + 16 = 0 0 ✓
2x2 = -5x +3 2x2 +5x -3 = 0 49 ✓
2
x - x = -2 2
x -x +2= 0 -7 ✓
2
1. x = - 3x - 12
2. x2 + 7x + 6 = 0

24
Note: Practice Personal Hygiene protocols at all times.
3. x2 + 6x - 10 = 0
4. +9 = -x2 + 6x
5. 2x2 = -7- 5x
6. 2x2 = -5x +3
7. 2x2 + 3x = 2
8 - 8x +16 = . - x2
9. 6x2 +6 = + 10x
10. +5 = . 5x2 - 10x

Rubric for Activity 2


5 4 3 2 1
Mathe- - Quadratic - Quadratic - Quadratic - Quadratic only one
matical equation equation was equation was equation was answer is
Concept was transformed transformed transformed correct
Used transformed correctly into correctly into incorrectly
correctly into its standard its standard into its
its standard form, the form , the standard
form, the value of the discriminant form , the
value of the discriminant is correct but discriminant
discriminant is correct but the value of and the
and the the nature of the nature of the
nature of the the roots is discriminant roots are
roots are incorrect or is incorrect or correct
correct no answer. no answer

Reflection
Every person has his unique personality. Same is true with the numbers.
Describe & tell something about your experience on this activity.

___________________________________________________________________________
___________________________________________________________________________
___________________________________________________________________________
___________________________________________________________________________
___________________________________________________________________________

References

1. Mathematics 9 LM, pp. 56-63


2. http://www.analyzemath.com/Equation/Quadratic-1.html
3. https://byjus.com/maths/nature-of-roots-quadratic/
25
Note: Practice Personal Hygiene protocols at all times.
Answer Key

Activity 1. Look then Fill Me Up


Quadratic Equation Quadratic Equation Value of
in Standard Form b2 – 4ac
(ax2 + bx + c = 0 ) (discriminant)
1. x2 + 3x + 12 = 0 x2 + 3x + 12 = 0 -39
2. x2 - 6x +9 = 0 x2 - 6x +9 = 0 0
3. 2x2 - 5x -3 = 0 2x2 - 5x -3 = 0 49
4. 2x2 + 3x -2 = 0 2x2 + 3x -2 = 0 25
2 2
5. x - 8x +16 = 0 x - 8x +16 = 0 0
2 2
6.3x = -2x -5 3x + 2x +5 = 0 -56
7. 3x2 - 12 = 7x 3x2 – 7x – 12 = 0 193
8. 2x – 6x = -12
2
2x – 6x + 12 = 0
2
-60
9. 5x2 = -2x - 5 5x2 + 2x + 5 = 0 -96
2 2
10. 5x = 5 5x - 5 = 0 100

Answer Key

Activity 2. Discriminant Ko Nature Mo?


Nature of the roots
Quadratic Equations ax2 + bx + c = 0 Value of two two two
b2 – 4ac equal unequal unequal
(discri- real real roots and
minant) roots imaginary
roots
1. x2 = - 3x - 12 1. x2 + 3x + 12 = 0 -39 /
2. x2 + 7x + 6 = 0 2. x2 + 7x + 6 = 0 25 /
3. x2 + 6x - 10 = 0 3. x2 + 6x - 10 = 0 76 /
4. +9 = -x2 + 6x 4. x2 - 6x +9 = 0 0 /
5. 2x2 = -7- 5x 5. 2x2 + 5x + 7 = 0 -31 /
6. 2x2 = -5x +3 6. 2x2 + 5x -3 = 0 49 /
7. 2x2 + 3x = 2 7. 2x2 + 3x -2 = 0 25 /
8 - 4x +4 = . - x2 8. x2 - 4x +4 = 0 0 /
9. 6x2 +6 = + 10x 9. 6x2 - 10x +6 = 0 -44 /
10. +5 = -5x2 - 10x 10. 5x2 + 10x +5 = 0 0 /
Prepared by:
SALIVI G. ELARDE
Writer
26
Note: Practice Personal Hygiene protocols at all times.
MATHEMATICS GRADE 9
Name of Learner: ______________________________ Grade Level: _____________
Section: ______________________________________ Date: ___________________

LEARNING ACTIVITY SHEET


Describing the Relationship Between the Coefficient and the Roots of
Quadratic Equation

Background Information for Learners


You already have a knowledge on the roots of quadratic equation using different
methods. This knowledge will help you understand the sum and product of the roots of
quadratic equations.

A. Finding the Sum and Product of the Roots of Quadratic Equations


Example: Solve for the roots
𝑥 2 − 𝑥 − 12 = 0
by factoring Let
(𝑥 − 4)(𝑥 + 3) 𝑥1 and 𝑥2 are roots
4, -3 → roots
𝑥1 = 4, 𝑥2 = −3
Sum of the roots
𝑥1 + 𝑥2 = 4 + (−3) = 1
Product of the roots
𝑥1 ∙ 𝑥2 = (4)(−3) = −12

Note: Long method is applied but in the next examples, short and easy method will work.

Note: All quadratic equations should be in the form 𝑎𝑥 2 + 𝑏𝑥 + 𝑐=0

Consider these in getting the sum and product of the roots quadratic equation
𝑏 𝑏
Sum of the Roots = − 𝑎 or simply, get the additive inverse of 𝑎
𝑐
Product of the roots= 𝑎

Examples:
1. 2𝑥 (3𝑥 − 5) − 2 = 3
6𝑥 2 − 10𝑥 − 2 − 3 = 3 − 3 (Addition Property of Equality)
6𝑥 2 − 10𝑥 − 5 = 0
𝑎 = 6, 𝑏 = −10, 𝑐 = −5

27
Note: Practice Personal Hygiene protocols at all times.
Sum of the roots
𝑏 −10 10 𝟓
− = −( )= =
𝑎 6 6 𝟑
Product of the roots
𝑐 −5 𝟓
= = −𝟔
𝑎 6

2. 3𝑥 2 + 𝑥 − 2 = 0
𝑎 = 3, 𝑏 = 1, 𝑐 = −2
1
Sum of the roots (use Additive Inverse of b)= − 3
2
Product of roots = − 3

B. Determining the Quadratic Equation Given the Roots

To determine the quadratic equation given the roots, get their sum and product.
Examples:
1) [-2, 5] → sum of the roots → −2 + 5 = 𝟑
→ product of the roots → (−2)(5) = −𝟏𝟎
Substitute:
𝑥 2 − 𝑠𝑢𝑚 𝑥 + 𝑝𝑟𝑜𝑑𝑢𝑐𝑡 = 0
𝑥 2 − 3𝑥 + (−10) = 0
𝑥 2 − 3𝑥 − 10 = 0 → Quadratic Equation

3 3 −3+20 −𝟏𝟕
2) [− 5 , 4] → sum of the roots → − 5 + 4 = =
5 𝟓
3 −𝟏𝟐
→ product of the roots → (5) (4) = 𝟓
Substitute:
𝑥 2 − 𝑠𝑢𝑚 𝑥 + 𝑝𝑟𝑜𝑑𝑢𝑐𝑡 = 0
−17 −12
𝑥2 − ( )𝑥 + ( )=0
5 5
17 12
5 (𝑥 2 + ( 5 ) 𝑥 − ( 5 ) = 0) → Multiply the entire equation by 5
5𝑥 2 + 17𝑥 − 12 = 0 → Quadratic Equation
Remember this:
𝑥 2 − 𝑠𝑢𝑚 𝑥 + 𝑝𝑟𝑜𝑑𝑢𝑐𝑡 = 0

3)[−2, ±3√2 ] → sum of the roots → −2 + 3√2


(+) −2 − 3√2
__________________
−𝟒
→ product of the roots → (−2 + 3√2)( −2 − 3√2) Use FOIL
= 4 + 6√2 − 6√2 − 9(2)
= 4 − 18
= −𝟏𝟒
28
Note: Practice Personal Hygiene protocols at all times.
Substitute:
𝑥 2 − 𝑠𝑢𝑚 𝑥 + 𝑝𝑟𝑜𝑑𝑢𝑐𝑡 = 0
𝑥 2 − (−4)𝑥 + (−14) = 0
𝑥 2 + 4𝑥 − 14) = 0 → Quadratic Equation

Learning Competency with Code:


Describes the relationship between the coefficients and the roots of a quadratic equation
(M9AL-Ic-2)

Directions: Read, study carefully and understand the following activities. Answer with
honesty.

Activity 1: Identify Me!


Directions: Given the quadratic equation, identify the values of a, b, and c.

Quadratic Equation a b c
1. 𝑥 2 + 6𝑥 − 16 = 0
2. 4𝑥 2 − 5𝑥 = 0
3. 2𝑥 2 + 𝑥 − 7 = 0
4. 3𝑥 2 − 15𝑥 + 5 = 0
5. 2𝑥 2 = 3 − 𝑥

Rubrics for Scoring


(1-5) 1 point each for correct value of a, b, c

Activity 2: This is My Sum and this is My Product. Who am I?


Directions: Use the values of a, b, and c in the determining the sum and product of the roots of
quadratic equations.
Sum Product
1. 𝑥 2 + 6𝑥 − 16 = 0 __________________ __________________
2. 4𝑥 2 − 5𝑥 = 0 __________________ __________________
3. 2𝑥 2 + 𝑥 − 7 = 0 __________________ __________________
2
4. 3𝑥 − 15𝑥 + 5 = 0 __________________ __________________
5. 2𝑥 2 = 3 − 𝑥 __________________ __________________
2 2
6. 5𝑥 − 5 = 3𝑥 − 2𝑥 __________________ ___________________
7. 2𝑥 (𝑥 − 1) + 5 = 0 __________________ ___________________
2
8. 3𝑥 − 𝑥 (𝑥 − 1) = 3 __________________ ___________________
9. (𝑥 + 5)2 = 0 __________________ ___________________
10. (𝑥 + 3)(𝑥 − 1) = 5 __________________ ___________________

Rubrics for Scoring


(1-5) 1 point each for correct answer 0.5 each for incorrect answer
(6-10) 2 points each for correct answer 1 point each for incorrect answer

29
Note: Practice Personal Hygiene protocols at all times.
Activity 3: Here are the roots. Where is the trunk?
Directions: Write the quadratic equations in the form 𝑎𝑥 2 + 𝑏𝑥 + 𝑐 = 0 given the following
roots. Show complete solutions.

Sum of the Roots Product of the Roots Quadratic Equation


1. [-5, 6] ______________ ________________ ________________
2. [12, -3] ______________ ________________ ________________
3. [0, -15] ______________ ________________ ________________
4. [15, -3] ______________ ________________ ________________
1
5. [2, − 2] ______________ ________________ ________________
3
6. [− 5 , −1] ______________ ________________ ________________
1 3
7. [ , − ] ______________ ________________ ________________
2 4
8. [±3√2] ______________ ________________ ________________
9. [±√3] ______________ ________________ ________________
10. [−1, ±4√5] ______________ ________________ ________________

Rubrics for Scoring


1 point each for sum and product 0.5 each for incorrect answer
2 points each for quadratic equation 1 point each for incorrect answer

Reflection
Roots of quadratic equations are very essential when associated with trees. Describe
how they are related to real-life situations.
___________________________________________________________________________
___________________________________________________________________________
___________________________________________________________________________
___________________________________________________________________________
___________________________________________________________________________

Reference
Mathematics Learner’s Material 9 First Edition, 2004 Department of Education, pp. 66- 72

30
Note: Practice Personal Hygiene protocols at all times.
ANSWER KEY
Activity 1: Identify Me!
Quadratic Equation a b C
2
1. 𝑥 + 6𝑥 − 16 = 0 1 6 -6
2. 4𝑥 2 − 5𝑥 = 0 4 -5 0
3. 2𝑥 2 + 𝑥 − 7 = 0 2 1 -7
2
4. 3𝑥 − 15𝑥 + 5 = 0 3 -15 5
5. 2𝑥 2 = 3 − 𝑥 2 1 -3

Activity 2: This is My Sum and this is My Product. Who am I?


Sum Product
1. 𝑥 2 + 6𝑥 − 16 = 0 ________-6________ ________-16_______
5
2. 4𝑥 2 − 5𝑥 = 0 _________4________ ________0_________
1 1
3. 2𝑥 2 + 𝑥 − 7 = 0 ________− 2________ _______− 2________
5
4. 3𝑥 2 − 15𝑥 + 5 = 0 _________5________ _________ ________
3
2 1 3
5. 2𝑥 = 3 − 𝑥 ________− ________ ________− ________
2 2
2 2 5
6. 5𝑥 − 5 = 3𝑥 − 2𝑥 ________-1_________ ________− 2________
5
7. 2𝑥 (𝑥 − 1) + 5 = 0 ________1_________ __________2________
1 3
8. 3𝑥 2 − 𝑥 (𝑥 − 1) = 3 ________− 2________ ________− 2________
9. (𝑥 + 5)2 = 0 ________-10________ _________25________
10. (𝑥 + 3)(𝑥 − 1) = 5 _________-1________ __________-8________

Activity 3: Here are the roots. Where is the trunk?


Sum of the Roots Product of the Roots Quadratic Equation
1. [-5, 6] ______1_______ _____-30______ 𝑥 2 − 𝑥 − 30 = 0
2. [12, -3] _____-15______ _____36_______ 𝑥 2 + 15𝑥 + 36 = 0
3. [0, -15] _____-15______ ______0_______ 𝑥 2 + 15𝑥 = 0
4. [15, -3] _____12_______ _____-45______ 𝑥 2 − 12𝑥 − 45 = 0
1 3
5. [2, − 2] _______2______ _____-1_______ 2𝑥 2 − 3𝑥 − 2 = 0
3 8 3
6. [− 5 , −1] ______− 5_____ ______5_______ 5𝑥 2 + 8𝑥 + 3 = 0
1 3 1 3
7. [2 , − 4] ______− 4_____ ______− 8_____ 8𝑥 2 + 2𝑥 − 3 = 0
8. [±3√2] _______0______ _____-18______ 𝑥 2 − 18 = 0
9. [2 ± √3] ______4_______ ______1_______ 𝑥 2 − 4𝑥 + 1 = 0
10. [−1 ± 4√5] ______-2_______ _____-79______ 𝑥 2 + 2𝑥 − 79 = 0
Prepared by:

AGNES M. TALDE
Writer

31
Note: Practice Personal Hygiene protocols at all times.
MATHEMATICS GRADE 9
Name: ________________________________ Grade Level:___________________
Section: ______________________________ Date: ________________________

LEARNING ACTIVITY SHEET


Solving Equations Transformable to Quadratic Equations (Including
Rational Algebraic Equations)

Background Information for Learners

This lesson aims to help you understand the steps on how to solve equation
transformable to quadratic equations which includes rational algebraic equations. From your
previous year level you have learned that quadratic equations are in the form 𝑎𝑥 2 +bx+c=0. In
this lesson an equation will be given then you will be required to transform it to its standard
form 𝑎𝑥 2 +bx+c=0 then solve for its values.
Let us have some examples.

Example 1.
x(x+5)=14
(x)(x) +(x)(5) = 14 (Transform into its standard form then solve for the value of x)
𝑥 2 + 5x= 14. (Apply the distributive property for multiplication)
𝑥 2 + 5x-14= 14-14 (Addition Property of Equality)
𝑥 2 + 5x -14 =0
(x+7)(x-2)=0 (Use factoring)

Solve for the value of x by addition property. Equate the factors to zero.
x+7=0
x+7=0-7
x= -7
x-2=0
x-2=0+2
x= 2
Therefore x=-7 and x=2

Example 2.
2x(x-2) = 3 (Transform in standard form then solve for the value of x)
(2x)(x) -(2x)(2) = 3 (Apply distributive property for multiplication)
2𝑥 2 -4x= 3
2𝑥 2 -4x-3= 3-3 (Using Addition Property of Equality)
2𝑥 2 - 4x -3 =0

32
Note: Practice Personal Hygiene protocols at all times.
Solve for the quadratic equation 2𝑥 2 - 4x -3 =0 using the Quadratic Formula.
−𝑏±√𝑏2 −4𝑎𝑐
Quadratic Formula: 𝑥 = a= 2, b=-4 and c=-3
2𝑎
−4±√(4)2−4(2)(−3)
𝑥= 2(2)
−4±√16+24
𝑥= 4
−𝟒±√𝟒𝟎
𝒙= 𝟏𝟒
−𝟒±𝟐√𝟏𝟎 −𝟐±√𝟏𝟎
𝒙= or 𝒙 =
𝟏𝟒 𝟕

(Note: You can either use factoring or apply the quadratic formula to solve for x.)

Example 3.
x(x+1)= 3(x+1) (Transform into its standard form then solve for the value of x)
(x)(x)+(x)(1)= (3)(x)+(3)(1) (Apply distributive property for multiplication)
𝑥 2 + x= 3x+3
𝑥 2 + x-3x-3= 3x+3 -3x-3 (Use Addition Property of Equality)
𝑥 2 + x -3x-3 =0 (Combine like terms)
𝑥 2 -2x-3 =0

Solve for the quadratic equation 𝑥 2 -2x-3 =0 using factoring.


By factoring it will become (x-3)(x+1)=0.

To solve for the value of x, use addition property and equate the factors to zero.
x-3=0
x-3=0+3
x= 3
x+1=0
x+1=0 - 1
x = -1
Therefore x=3 and x=-1

Learning Competency with Code: Solves equations transformable to quadratic equations


(including rational algebraic equations) (M9AL-Ic-d-1)

Directions: Read, study carefully and understand the following activities. Answer with
honesty.

33
Note: Practice Personal Hygiene protocols at all times.
Activity

Direction/ Instructions:
Solve the following equations by transforming to standard form 𝑎𝑥 2 +bx +c = 0. Solve for the
values of x using factoring or Quadratic formula.

1. x(x+3) =4

2. x(x+3) =10

3. x(2x+1)= 2(x-1)

4. (𝑥 + 5)2 + (𝑥 − 2)2=37

5. (𝑥 + 6)2 =15

Reflection

This topic makes you realize that we can simplify things if we are going to take it one
at a time.
Write down below your reflections from this activity.
___________________________________________________________________________
___________________________________________________________________________
___________________________________________________________________________
___________________________________________________________________________
___________________________________________________________________________

References
1. RD Sharma, Mathematics for Class 9

2. Mathematics 9 Learner’s Material


Department of Education
Republic of the Philippines

34
Note: Practice Personal Hygiene protocols at all times.
Key to Correction
Answer
1. x(x+3) =4 -4, -1

2. x(x+3) =10 -5, 2

1±√−15
3. x(2x+1)= 2(x-1) 4

4. (𝑥 + 5)2 + (𝑥 − 2)2=37 -4, 1

5. (𝑠 + 6)2 =15 −6 ± √15

Prepared by:

RIZETTE A. MONTERO
Writer

35
Note: Practice Personal Hygiene protocols at all times.
MATHEMATICS GRADE 9
Name: ________________________________ Grade Level:___________________
Section: ______________________________ Date: ________________________

LEARNING ACTIVITY SHEET


Solving Equations Transformable to Quadratic Equations (Including
Rational Algebraic Equations)

Background Information for Learners


This topic is still under solving equations transformable to quadratic equations which
will be focusing on rational algebraic expressions. From previous topics you had learned that
a rational expressions are expressions that have polynomial/s on the numerator or denominator.
This discussion helps you understand steps on how to transform rational algebraic expression
to standard form of quadratic equation.

Let us review Rational algebraic expressions.


Determine whether the expression is rational or Not Rational.

𝑥−6 6
1. + =0 Rational
𝑥 𝑥

2. 3x +4 =12 Not Rational

1 3
3. 2𝑛 + =10 Rational
2𝑛

4. 𝑥 2 -4x + 3=0 Not Rational

Since you already know the definition of rational algebraic expression. Let us have an
example on how to solve rational expression transformable to standard form of quadratic
equation.
Example 1:
Transform the following to the standard form of quadratic equation then solve for x.
1 3
− = 7.
𝑥2 𝑥

Determine the Least Common Denominator (LCD) of the given equation.


The LCD is ( 𝑥 2 ).
1 3
( 𝑥 2 ) ( 𝑥2 )- ( 𝑥 2)( 𝑥) = ( 𝑥 2 )(7) (Multiply the LCD to the given equation)
1 3
( 𝑥 2 ) ( 𝑥2 )- ( 𝑥 2)( 𝑥) = ( 𝑥 2 )(7)
(1) - (x)(3) = (𝑥 2 )(7)
1-3x=7𝑥 2

36
Note: Practice Personal Hygiene protocols at all times.
Arrange the equation to standard form of quadratic equation which is 𝑎𝑥 2 +bx+c=0 and
combine similar terms. The new equation will be
7𝑥 2 +3x-1=0

−𝑏±√𝑏2 −4𝑎𝑐
Quadratic Formula: 𝑥 = (Use quadratic formula to solve for the values of x)
2𝑎
a= 7, b=3 and c=-1
−3±√(3)2−4(7)(−1)
𝑥= 2(7)
−3±√9+28
𝑥= 14
−𝟑±√𝟑𝟕
𝒙= = Final answer
𝟏𝟒

Example 2:
4 𝑥
+ = -2
𝑥−3 2
Determine the Least Common Denominator (LCD) of the given equation. The LCD is
(x-3)(2).
4 𝑥
(x-3)(2)( 𝑥−3)+ (x-3)(2)(2 )= (x-3)(2)(-2) (Multiply the LCD to the given equation)
4 𝑥
(x-3)(2)( 𝑥−3)+ (x-3)(2)(2 )= (x-3)(2)(-2)
(2)(4) + (x-3)(x) =(x-3)(2)(-2)
8+ 𝑥 2 - 3x = -4x+12

Arrange the equation into standard form of quadratic equation which is 𝑎𝑥 2 +bx+c=0 and
combine similar terms together. The new equation will be
𝑥 2 -3x+4x-12+8=0
𝑥 2 + x - 4=0 (Express in standard form)
−𝑏±√𝑏2 −4𝑎𝑐
Quadratic Formula: 𝑥 = (Use the quadratic formula to solve for the values of x)
2𝑎
a= 1, b=1 and c=-4
−1±√(1)2−4(1)(−4)
𝑥= 2(1)
−1±√1+16
𝑥= 2
−𝟏±√𝟏𝟕
𝒙= = Final answer
𝟐

Learning Competency with Code:


Solves equations transformable to quadratic equations (including rational algebraic
equations) (M9AL-Ic-d-1)

Directions: Read, study carefully and understand the following activities. Answer with
honesty.

37
Note: Practice Personal Hygiene protocols at all times.
Activity 1: What Am I!
Direction/ Instructions: Identify if the given expression is Rational or Not. Write Rational
expression if it is a rational expression and Not Rational expression if it is not a rational
expression. Write your answer on the space provided.

1 4𝑥
1. + =1 ___________________
3𝑥 6
2 8
2. 𝑥 -3x+4x=4 ___________________
5 𝑥 2
3. . + 9= 4 ___________________
𝑥
2
4. 𝑥 -3x+4x=0 ___________________
4𝑥
5. 3+ 6 =1 ___________________

Activity 2 Help Me Look For My X


Directions: Solve the following equations by transforming it to 𝑎𝑥 2 +bx+c=0. Solve for the
value of x using factoring or Quadratic formula.

2 4
1. −𝑥=3
𝑥2

2 𝑥2 5𝑥
2. + = 10
5 4

1 4𝑥
3. 3𝑥+ 6 =1

6 𝑥−3
4. + =2
𝑥 4

1 𝑥 2
5. + =
𝑥 6 3

Rubrics for the Activity


Criteria 5 4 3 2 1 Score
Understand Student Student was Student Students Student
the problem was able to not able to missed was not able was not
understand understand special to able to
special some of the factors of understand understand
factors to minor the problem almost all of the
come up factors to the special problem
with the come up factors
solution. with the
solution
Apply Student Student Students Student was Student
proper was able to misses was not able not able to was not
procedures solve the some points to come up what was able to
problem such as with the asked but
38
Note: Practice Personal Hygiene protocols at all times.
in the accurately units or correct final show some answer the
problem. and proper way answer but correct problem.
completely of rounding. answer solutions.
minor
computation
Overall

Reflection
This topic makes you realize that complicated things can be simplified.
Write down below your reflections on the activity
___________________________________________________________________________
___________________________________________________________________________
___________________________________________________________________________
___________________________________________________________________________
___________________________________________________________________________

References
1. RD Sharma, Mathematics for Class 9

2. Mathematics 9 Learner’s Material


Department of Education
Republic of the Philippines

39
Note: Practice Personal Hygiene protocols at all times.
ANSWER KEY
Activity 1 (What am I !)

1 4𝑥
1. 3𝑥+ 6 =1 Rational Expression
8
2. 𝑥 2 -3x+4x= Not a Rational Expression
4
5 𝑥 2
3. . + 9= 4 Rational Expression
𝑥
2
4. 𝑥 -3x+4x=0 Not a Rational Expression
4𝑥
5. 3+ 6 =1 Rational Expression

Activity 2: Help me look for my x.


Answer
2 4 −2±√10
1. −𝑥=3 𝑥=
𝑥2 3

2 𝑥2 5𝑥 −25±√−5775
2. + = 10 𝑥=
5 4 16

1 4𝑥 1
3. 3𝑥+ 6 =1 𝑥 = 2,1

6 𝑥
4. + =2 𝑥 = 8, 3
𝑥 4

1 𝑥 2
5. +6 = 3 𝑥 = 2 ± √−2
𝑥

Prepared by:

RIZETTE A. MONTERO
Writer

40
Note: Practice Personal Hygiene protocols at all times.
MATHEMATICS GRADE 9
Name: ________________________________ Grade Level:___________________
Section: ______________________________ Date: ________________________

LEARNING ACTIVITY SHEET


Solving Problems Involving Quadratic Equations and Rational
Algebraic Equations

Background Information for Learners

From the previous topic you had learned about quadratic equations. In this lesson we
will still be applying quadratic equation however there is a need for you to analyze the problems
to come up with the correct representation. In this topic you need to understand some
mathematical expressions in the formulation of the representation.
a. 2 Consecutive even /Odd Numbers :
First number is represented by x
Second number is represented by x+2

b. 2 Consecutive numbers
First number is represented by x
Second number is represented by x+1

c. Pythagorean Theorem = represented by 𝑎2 + 𝑏2 = 𝑐 2 , it is used to solve missing sides of a


right triangle.

d. Areas of polygon
Area
Triangle 1
A= 2 (𝑏ℎ), where b- base, h-height
Square A= 𝑠 2 , where s- side
Rectangle A= 𝑙𝑤, where l-length, w-width

To further explain the topic let us have examples.

Example 1.
The product of two consecutive positive integers is 5 more than three times the larger.
Find the integers.
1st step: Formulate representation
x → first integer
x+1 → second integer
The representation of the product of the two positive integers will be

41
Note: Practice Personal Hygiene protocols at all times.
x(x+1), then it says three times the larger so it will become 3(x+1) +5

x(x+1)= 3(x+1) +5 (Formulate equation)


x(x)+(x)(1)= 3(x)+(3)(1) +5 (Distributive Property)
𝑥 2 +x =3x+3+5
𝑥 2 +x =3x+8 (Combine like terms)
𝑥 2 +x-3x -8=3x+8-3x -8 (Addition Property of Equality)
𝑥 2 -2x -8=0
(x-4)(x+2)=0 (Factor the quadratic equation)
Therefore the values of x are 4 and -2.

Note: Since the problem requires positive integer therefore the value of x is 4

Going back to the representation:

The first integer is x which is equal to 4 then the second integer is (x+1) and 4 + 1=5.Therefore
the two positive integers are 4 and 5.

Example 2.
A rectangular building measuring 30m by 25m is going to have its area increased by
2
200𝑚 by adding a strip of uniform width to all four sides. Determine the width of the strip.
For the diagram of this situation, we have a smaller rectangle (the original building) surrounded
by a larger rectangle (the new building).We will let x represent the unknown width of the strip.
If the dimension of the original building were 30 m by 25 m , then the dimension of the new
building will be 30 +2x by 25 by 2x.
x
25+2x

30+2x

If the area of the original building was 750𝑚2 , then the area of the new building will be 950𝑚2 .
Since the new building is still a rectangle therefore the area is length multiplied by width,
A=l x w.
950=(30+2x)(25+2x)
950=750 +60x +50x+4𝑥 2 (Distributive Property)
2
0=4𝑥 + 110x -200
4𝑥 2 + 110x -200=0 (arrange the equation into its standard form)
2
2𝑥 + 55x -100=0 (Reduce by 2)

By applying the quadratic formula,


−𝑏±√𝑏2 −4𝑎𝑐
Quadratic Formula: 𝑥 = 2𝑎

42
Note: Practice Personal Hygiene protocols at all times.
a= 2, b=55 and c=-100
−55±√(55)2−4(2)(−100)
𝑥= 2(2)
−55±√3025+800
𝑥= 4
−55±√3825
𝑥= 4
−55±61.85
𝑥= 4
6.85
𝑥= 4
𝑥 = 1. 71 Final Answer

The width of the strip is 1.7m.


Note: Disregard the -29.2 since the width cannot be negative.

Example 3:
The hypotenuse of a right triangle is 3 cm longer that one of the legs, and 6 cm longer that the
other leg, find the length of the three sides of the triangle.

Let x represents the length of the hypotenuse, the longer leg would be x-3, and the shorter leg
would be x-6.

x-6 x

x-3
The relationship between the three sides of a right triangle is described by the Pythagorean
Theorem
𝑎2 + 𝑏 2 = 𝑐 2
(𝑥 − 3)2 + (𝑥 − 6)2 = 𝑥 2
(𝑥 − 3)2
(𝑥 − 3)(𝑥 − 3) (Distributive Property of Multiplication)
2
𝑥 -6x+9

(𝑥 − 6)2
(x-6)(x-6) Square the binomial
2
𝑥 -12x+36
𝑥 2 -6x+9+ 𝑥 2 -12x+36= 𝑥 2
𝑥 2 -18x+45=0 (Combine like terms)
(x-15)(x-3) =0 (Factor out the quadratic equation)
x = 3 and 15.
Note: Disregard 3 because it will generate leg length of 0 cm and -3 cm.
Therefore the value of longest is 15cm, longer leg is 12 cm and the shorter leg is 9cm.
Example 4:

43
Note: Practice Personal Hygiene protocols at all times.
The length of a rectangle is 3 meters more than twice the width, and the area of the rectangle
is 35𝑚2 . Find the dimensions of the rectangle.

2w+3
Representation
w → width
2w+3 → length

Area of a rectangle is A=l x w


Therefore,
35= (2w+3)(w)
35= 2𝑤 2 + 3w (Distributive Property of Multiplication)
35 −2𝑤 2 - 3w = 2𝑤 2 + 3w- 2𝑤 2 - 3w (Addition Property of Equality)
2𝑤 2 + 3w-35=0
(w+5)(2w-7)=0 (Factor the Quadratic equation)
(w+5)=0 , w=-5 (Equate the equation to 0)
(2w-7)=0 w=3.5

Note : Disregard negative value because we cannot have a negative length.


Therefore,
w=3.5 → width
2w+3
2(3.5)+3= 7+3=10 m → length
To solve for the other side, apply the area of rectangle.

Checking:
A=l x w
35𝑚2 = (10m) (3.5 m)
35𝑚2 =35𝑚2

Learning Competency with Code: Solves problems involving quadratic equations and
rational algebraic equations (M9AL-Ie-1)

Directions: Read, study carefully and understand the following activities. Answer with
honesty.

44
Note: Practice Personal Hygiene protocols at all times.
Activity (Find Me!)

Direction/ Instructions:
Solve the following problems accurately. Show your representation, equation and solution
completely and neatly.

1. Find 2 consecutive positive odd integers whose product is 63.

2. The product of two numbers is 72. The larger number is 4 more than 3 times the smaller
number. Find them.

3. Find 2 consecutive positive integers whose product is 132.

4. The area of a rectangle is 65𝑚2 and the length is 3 meter less than twice the width. Find the
dimensions.

5. The length of a rectangle is 5m more than twice its width, and the area is 75𝑚2 . Find the
dimensions of the rectangle.

6. The area of the square is 80𝑚2 . Find the length of its side.

45
Note: Practice Personal Hygiene protocols at all times.
7. The product of the ages of a father and his son is 576. If the father is 18 years younger than
5 times his son, how old is each?

8. The hypotenuse of a right triangle is 4 m longer than the shorter leg and 2 m longer than the
longer leg. Find the lengths of the sides.

Rubrics for the Activity


Criteria 5 4 3 2 1 Score
Understand Student Student was Student Students was Student
the problem was able to not able to missed not able to was not
understand understand special understand able to
special some of the factors of the almost all of understand
factors to minor factors problem the special the
come up to come up factors problem
with the with the
solution. solution
Representation Students Student Student Students was Student
of the problem was able to missed minor missed a not able to was not
represent representation major represent able to
completely representation completely the represent
the in his solution major and the
problem minor problem.
representation.
Apply proper Student Student Students was Student was Student
procedures in was able to misses some not able to not able to was not
the problem. solve the points such as come up with what was able to
problem units or the correct asked but answer the
accurately proper way of final answer show some problem.
and rounding. but answer correct
completely minor solutions.
computation
Overall
Reflection
This topic makes you realize that we need to analyze problems intensively to come up
with the correct solution. Write your reflections below from the lesson you learned.
___________________________________________________________________________
___________________________________________________________________________
___________________________________________________________________________
___________________________________________________________________________
___________________________________________________________________________

References
1. RD Sharma, Mathematics for Class 9
46
Note: Practice Personal Hygiene protocols at all times.
2. Mathematics 9 Learner’s Material
Department of Education
Key to Correction

Activity (Find Me!)


1. Find 2 consecutive positive odd integers whose product is 63.
Answer: 7 and 9
2. The product of two numbers is 72. The larger number is 4 more than 3 times the smaller
number. Find them.
Answer: 4 and 18
3. Find 2 consecutive positive integers whose product is 132.
Answer: 11 and 12
4. The area of a rectangle is 65𝑚2 and the length is 3 meter less than twice the width. Find the
dimensions.
Answer: width: 5 m
length: 13 m
5. The length of a rectangle is 5m more than twice its width, and the area is 75𝑚2 . Find the
dimensions of the rectangle.
Answer: width: 5 m
length: 15 m
6. The area of the square is 80𝑚2 . Find the length of its side.
Answer: 𝟒√𝟓 𝒎
7. The product of the ages of a father and his son is 576. If the father is 18 years younger than
5 times his son, how old is each?
Answer: son: 12 years old
father: 48 years old
8. The hypotenuse of a right triangle is 4 m longer than the shorter leg and 2 m longer than the
longer leg. Find the lengths of the sides.
Answer: shorter leg: 6m
longer leg: 8m
hypotenuse: 10 m

Prepared by:

RIZETTE A. MONTERO
Writer

47
Note: Practice Personal Hygiene protocols at all times.
MATHEMATICS GRADE 9
Name of Learner: ______________________________ Grade Level: _____________
Section: ______________________________________ Date: ___________________

LEARNING ACTIVITY SHEET


Illustrating Quadratic Inequalities

Background Information for Learners

Quadratic inequality is an inequality that contains a polynomial of degree 2 and can be


written in the forms 𝑎𝑥 2 + 𝑏𝑥 + 𝑐 > 0 (read as 𝑎𝑥 2 + 𝑏𝑥 + 𝑐 greater than 0)
𝑎𝑥 2 + 𝑏𝑥 + 𝑐 ≥ 0 (read as 𝑎𝑥 2 + 𝑏𝑥 + 𝑐 greater than or equal to 0)
𝑎𝑥 2 + 𝑏𝑥 + 𝑐 < 0 (read as 𝑎𝑥 2 + 𝑏𝑥 + 𝑐 less than 0)
𝑎𝑥 2 + 𝑏𝑥 + 𝑐 ≤ 0 (read as 𝑎𝑥 2 + 𝑏𝑥 + 𝑐 less than or equal to 0)

Examples: 1. 𝑥 2 − 3𝑥 + 2 > 0 is read as 𝑥 2 − 3𝑥 + 2 is greater than zero


2. 𝑥 2 − 3𝑥 + 2 ≥ 0 is read as 𝑥 2 + 3𝑥 + 2 is greater than or equal to zero

Remember this in illustrating Quadrating Inequalities:


greater than the solutions go in opposite Unshaded circle (the
> directions on the number number is not included in
line the solutions)
greater than or the solutions go in opposite Closed/Shaded circle (the
≥ equal to directions on the number number is included in the
line solutions)
less than the solutions go toward each Open circle (the number is
< other on the number line not included in the
solutions)
less than or the solutions go toward each Closed/Shaded circle (the
≤ equal to other on the number line number is included in the
solutions)

Writing and illustrating mathematical notation on quadratic inequalities


Examples:
Write the correct Mathematical Notation based on the given illustrations on quadratic
inequalities.

1. Given Answer in Mathematical Notation


x
−3 > 𝑥 > 4 → x is less than -3 but greater than 4
-3 4
2.
x
−5 < 𝑥 < 2 → x is greater than -5 but less than 2
-5 2
48
Note: Practice Personal Hygiene protocols at all times.
3.
x −1 ≥ 𝑥 ≥ 5 → x is less than or equal to -1 but
-1 5 greater than or equal to -1

4.
x 0 ≤ 𝑥 ≤ 7 → x is greater than or equal to 0 but
0 7 less than or equal to 7

Learning Competency with Code:


Illustrates quadratic inequalities (M9AL-If-1)

Directions: Read, study carefully and understand the following activities. Answer with
honesty.

ACTIVITY 1: Explore!
Directions: Write the correct Mathematical Notation based on the given illustrations on
quadratic inequalities.
Given Answer
1.
_________________________________
-7 4
2.
_________________________________
0 10
3.
_________________________________
-12 -3
4. _________________________________
-8 8

5. _________________________________
-4 12
6. _________________________________
-15 0
7. _________________________________
-3 15
8. _________________________________
4 15

9. _________________________________
1 1
-4 2
10. 2 _________________________________

1 1
10
3 5

49
Note: Practice Personal Hygiene protocols at all times.
Activity 2: Am I a Quadratic Inequality?
Directions: Determine whether each mathematical sentence is a quadratic inequality
or not. Write ( ) if it is a quadratic inequality and (X) if NOT.
Answer
1. 𝑥 2 + 2𝑥 > 0 ____________________
2. 5𝑥 + 20 ≤ 0 ____________________
3. 3𝑎2 − 𝑎 = 1 ____________________
4. 𝑥 (𝑥 − 1) > 5 ____________________
5. (2𝑚 − 5)(𝑚 + 1) < 3 ____________________
6. 4ℎ(ℎ − 7) ≥ 2ℎ(1 + 2ℎ) ____________________
7. 15 − 2𝑥 = 3𝑥 2 ____________________
8. 𝑥 2 − 1 < 𝑥 − 1 ____________________
9. 4𝑝2 ≤ 1 ____________________
10. 5𝑥 (1 + 𝑥 ) ≥ 5𝑥 2 − 3𝑥 + 1 ____________________
Reflection

The symbols of greater than and less than show that two expressions are different. How
would you describe the illustrations on this activity?

___________________________________________________________________________
___________________________________________________________________________
___________________________________________________________________________
___________________________________________________________________________
___________________________________________________________________________
Reference
Mathematics Learner’s Material 9 First Edition, 2004 Department of Education, pp. 114-115

ANSWER KEY
Activity 1: Explore!
Given Answer
1.
−7 > 𝑥 > 4
-7 4
2.
0 > 𝑥 > 10
0 10 50
Note: Practice Personal Hygiene protocols at all times.
3.
−12 < 𝑥 < −3
-12 -3
4. −4 < 𝑥 < 12
-4 12

5. −8 ≥ 𝑥 ≥ 8
-8 8
6. −15 ≥ 𝑥 ≥ 0
-15 0

7. −3 ≤ 𝑥 ≤ 15
-3 15
8. 4 ≤ 𝑥 ≤ 15
4 15
1 1
9. 1
−4 2 > 𝑥 > 2
1
-4
2 2
1 1
10. ≤ 𝑥 ≤ 10 5
3
1 1
10
3 5

Activity 2: Am I a Quadratic Inequality?


1.
2. X
3. X
4.
5.
6. X
7. X
8.
9.
10. X

Prepared by:

CRIZALYN JOYCE Q. BARTOLOME


Writer

51
Note: Practice Personal Hygiene protocols at all times.
MATHEMATICS GRADE 9
Name of Learner: ________________________________ Grade Level: _____________
Section: ________________________________________ Date: ___________________

LEARNING ACTIVITY SHEET


Solving Quadratic Inequalities in One Variable

Background for Learners

To solve a quadratic inequality in one variable, find the roots of its corresponding
equality, then illustrate in a number line.
Use factoring to solve for the roots in its corresponding equality.

A. Solving Quadratic Inequalities in One Variable


Examples:
1. 𝑥 2 + 2𝑥 − 15 > 0 → use quadratic trinomial factoring
(𝑥 − 3)(𝑥 + 5)
3, 5 → roots
Illustrations

-5 3
Note: The solutions go on opposite directions because of > as symbol of inequality. Arrows
represent the solutions.
Solutions:
𝒙 > 𝟑 and 𝒙 < −𝟓 or −𝟓 > 𝒙 > 𝟑

Checking:
Let 𝑥 = 4
Substitute 𝑥 2 + 2𝑥 − 15 > 0 , Therefore,
(4)2 + 2(4) − 15 > 0
16 + 8 − 15 > 0
24 − 15 > 0
9 > 0 TRUE
Therefore, 4 is a solution.

2. 𝑥 2 − 6𝑥 < 0 → by Common Monomial Factoring


𝑥 (𝑥 − 6)
0, 6 → roots

Illustration:
Note: The solutions go toward each other
because of < as symbol of inequality.
0 6
52
Note: Practice Personal Hygiene protocols at all times.
Solutions:
𝑥 > 0, 𝑥 < 6 or 6 > 𝑥 > 0 (1, 2, 3, 4, 5)

Checking:
Let 𝑥 = 2
Substitute 𝑥 2 − 6𝑥 < 0
(2)2 − 6(2) < 0
4 − 12 < 0
−8 < 0 TRUE
Therefore, 2 is a solution.

3. 𝑥 2 − 4𝑥 − 12 ≥ 0 → by Quadratic Trinomial Factoring


(𝑥 − 6)(𝑥 + 2)
-2, 6 → roots
Illustration:
Note: -2 and 6 are included as solutions
because the circles are shaded.
-2 6
Solutions:
𝒙 ≥ 𝟔, 𝒙 ≤ −𝟐 or −𝟐 ≥ 𝒙 ≥ 𝟔
Checking:
Let 𝑥 = 6
Substitute 𝑥 2 − 4𝑥 − 12 ≥ 0
(6)2 − 4(6) − 12 ≥ 0
36 − 24 − 12 ≥ 0
36 − 36 ≥ 0
0 ≥ 0 TRUE
Therefore, 6 is a solution.
B. Quadratic Inequalities in Two Variables
These inequalities can be written in the following forms:
𝑦 > 𝑎𝑥 2 + 𝑏𝑥 + 𝑐, 𝑦 ≥ 𝑎𝑥 2 + 𝑏𝑥 + 𝑐
𝑦 < 𝑎𝑥 2 + 𝑏𝑥 + 𝑐, 𝑦 ≤ 𝑎𝑥 2 + 𝑏𝑥 + 𝑐
In this topic, you will determine if a point is a solution of a given quadratic inequality
in two variables.
Example 1. Determine whether or not each given point is a solution of the given quadratic
inequality 𝑦 < 2𝑥 2 − 𝑥 + 3
a) Point (-3, 1), x = -3, y = 1
Substitute in 𝑦 < 2𝑥 2 − 𝑥 + 3
1 < 2(−3)2 − (−3) + 3
1 < 2(9) − 3 + 3
1 < 18 + 6
1 < 24 TRUE
Therefore, (-3, 1) is a solution.
53
Note: Practice Personal Hygiene protocols at all times.
b) Point (2, 10), x = 2, y =10
Substitute in 𝑦 < 2𝑥 2 − 𝑥 + 3
10 < 2(2)2 − (2) + 3
10 < 2(4) − 2 + 3
10 < 8 − 2 + 3
10 < 9 FALSE
Therefore, (2, 10) is NOT a solution.
Example 2: Determine whether or not each given point is a solution of the given quadratic
inequality 𝑦 ≥ 3𝑥 2 − 7
a) Point (1, -4), x = 1 , y = -4
Substitute in 𝑦 ≥ 3𝑥 2 − 7
−4 ≥ 3(1)2 − 7
−4 ≥ 3 − 7
−4 ≥ −4
−4 = −4 TRUE
Therefore, (1, -4) is a solution.
b) Point (-2, 12) x = 2, y = 12
Substitute in 𝑦 ≥ 3𝑥 2 − 7
12 ≥ 3(−2)2 − 7
12 ≥ 3(4) − 7
12 ≥ 12 − 7
12 ≥ 5 TRUE
Therefore, (-2, 12) is a solution.

Learning Competency with Code:


Solves quadratic inequalities (M9AL-If-2)

Directions: Read, study carefully and understand the following activities. Answer with
honesty

54
Note: Practice Personal Hygiene protocols at all times.
ACTIVITY 1: Describe My Solutions
Directions/Instructions: Solve for the solutions of the following quadratic inequalities by
showing your illustrations as basis in writing the solutions in Mathematical notation.

Illustration Solutions in
Mathematical Notation

1. 𝑥 2 − 3𝑥 > 0 _________________ __________________________

2. 𝑥 2 + 4𝑥 < 0 _________________ __________________________

3. 𝑥 2 − 2𝑥 − 8 > 0 _________________ __________________________

4. 𝑥 2 + 8𝑥 + 12 < 0 _________________ __________________________

5. 𝑥 2 − 𝑥 − 20 ≤ 0 _________________ __________________________

6. 𝑥 2 + 9𝑥 + 18 ≥ 0 _________________ __________________________

Rubrics for Scoring


2 points each → illustration
2 points each → solutions
1 point each for incorrect answers

Activity 2: Am I a Solution or Not?


Directions: Determine whether or not each of the following point is a solution of the quadratic
inequality. Justify your answer.
A. Given 𝑦 > 𝑥 2 − 5𝑥 + 1 (1 and 2)
Example: Justifications Answer (Solution/ Not)
(-2, 6) 6 > 15 Not
1. (-5, -8) ___________ ___________________
2. (-1, 15) ___________ ___________________
2
B. Given 𝑦 ≥ 𝑥 − 5𝑥 − 3 (3, 4 and 5)
3. (2, -9) ___________ ___________________
4. (-6, 10) ___________ ___________________
5. (1, 15) ___________ ___________________
55
Note: Practice Personal Hygiene protocols at all times.
C. Given 𝑦 < −2𝑥 2 + 5 (6 and 7)
6. (4, -12) ___________ ___________________
7. (-5, 0) ___________ ___________________
2
D. Given 𝑦 ≤ 𝑥 + 5𝑥 − 6 (8, 9 and 10)
8. (0, -8) ___________ ___________________
9. (3, 18) ___________ ___________________
10. (-8, 8) ___________ ___________________

Reflection
To trust the one you Love is a good gesture but to verify is better. Does this apply in this
activity? Relate.
___________________________________________________________________________
___________________________________________________________________________
___________________________________________________________________________
___________________________________________________________________________
___________________________________________________________________________
Reference

Mathematics Learner’s Material 9 First Edition, 2004 Department of Education, pp. 116-122

56
Note: Practice Personal Hygiene protocols at all times.
ANSWER KEY
Activity 1: Describe My Solutions
Illustration Solutions in
Mathematical Notation

1. 𝑥 2 − 3𝑥 > 0 0>𝑥>3
0 3

2. 𝑥 2 + 4𝑥 < 0 −4 < 𝑥 < 0


-4 0

3. 𝑥 2 − 2𝑥 − 8 > 0 −2 > 𝑥 > 4


-2 4

4. 𝑥 2 + 8𝑥 + 12 < 0 −6 < 𝑥 < −2


-6 -2

5. 𝑥 2 − 𝑥 − 20 ≤ 0 −4 ≤ 𝑥 ≤ 5
-4 5

6. 𝑥 2 + 9𝑥 + 18 ≥ 0 −6 ≥ 𝑥 ≥ −3
-6 -3
Activity 2:
A. Given 𝑦 > 𝑥 2 − 5𝑥 + 1 (1 and 2)
Example: Justifications Answer (Solution/ Not)
(-2, 6) 6 > 15 Not
1. (-5, -8) −8 > 51 Not
2. (-1, 15) 15 > 7 Solution
2
B. Given 𝑦 ≥ 𝑥 − 5𝑥 − 3 (3, 4 and 5)
3. (2, -9) −9 = −9 Solution
4. (-6, 10) 10 ≥ 63 Not
5. (1, 15) 15 ≥ 7 Solution
2
C. Given 𝑦 < −2𝑥 + 5 (6 and 7)
6. (4, -12) −12 < −27 Not
7. (-5, 0) 0<5 Solution
2
D. Given 𝑦 ≤ 𝑥 + 5𝑥 − 6 (8, 9 and 10)
8. (0, -8) −8 ≤ −6 Solution
9. (3, 18) 18 = 18 Solution
10. (-8, 8) 8 ≤ 18 Not

Prepared by:

CRIZALYN JOYCE Q. BARTOLOME


Writer
57
Note: Practice Personal Hygiene protocols at all times.
MATHEMATICS GRADE 9
Name of Learner: ________________________________ Grade Level: _____________
Section: ________________________________________ Date: ___________________

LEARNING ACTIVITY SHEET


Solving Problems Involving Quadratic Inequalities

Background Information for Learners


Quadratic Inequalities are mathematical sentences that can be written in any of the
following forms: 𝑎𝑥 2 + 𝑏𝑥 + 𝑐 > 0, 𝑎𝑥 2 + 𝑏𝑥 + 𝑐 < 0, 𝑥 2 + 𝑏𝑥 + 𝑐 ≥ 0, 𝑥 2 + 𝑏𝑥 + 𝑐 ≤
0.
In solving problems involving quadratic inequalities, the following must be considered:
1. Analyze and Represent the Problem
2. Write the mathematical inequality
3. Solve the Problem
4. Check
Inequality symbols should be represented correctly.
REMEMBER:
< less than, fewer than, up to
> greater than, more than, exceeds, in excess of
≤ Less than or equal to, at most, not more than
≥ Greater than or equal to, at least, not less than

EXAMPLE:
One leg of a right triangle is 7cm shorter than the other leg. How long should the shorter
leg be to ensure the hypotenuse is at least 13cm?
SOLUTION:
Inequality: (At least)→ ≥ c
a
Let 𝑥 → 𝑙𝑜𝑛𝑔𝑒𝑟 𝑙𝑒𝑔
𝑥 − 7 → 𝑠ℎ𝑜𝑟𝑡𝑒𝑟 𝑙𝑒𝑔 b

The given problem above is about right triangle, wherein Pythagorean Theorem is applied.
Pythagorean Theorem: 𝑎2 + 𝑏2 = 𝑐 2 ,
𝑤ℎ𝑒𝑟𝑒 𝑎 𝑎𝑛𝑑 𝑏 𝑎𝑟𝑒 𝑡ℎ𝑒 𝑙𝑒𝑔𝑠 𝑎𝑛𝑑 𝑐 𝑖𝑠 𝑡ℎ𝑒 ℎ𝑦𝑝𝑜𝑡𝑒𝑛𝑢𝑠𝑒
Thus, 𝑎2 + 𝑏2 ≥ 𝑐 2
𝑥 2 + (𝑥 − 7)2 ≥ 132 (Substitution)
2 2
𝑥 + 𝑥 − 14𝑥 + 49 ≥ 169 (Simplify)
2𝑥 2 − 14𝑥 + 49 − 169 ≥ 0 (Combine like terms)
2𝑥 2 − 14𝑥 − 120 ≥ 0 (Simplify)
58
Note: Practice Personal Hygiene protocols at all times.
Since, the entire quadratic inequality had a common factor, we divide each term by the GCF,
thus,
2𝑥 2 14𝑥 120
− − ≥ 0 𝑡ℎ𝑒 𝑟𝑒𝑠𝑢𝑙𝑡 𝑖𝑠 (𝑥 2 − 7𝑥 − 60) ≥ 0
2 2 2
(𝑥 − 12)(𝑥 + 5) ≥ 0 (Factor)
Therefore, 𝒙 ≥ 𝟏𝟐, 𝒙 ≤ −𝟓 (𝒓𝒐𝒐𝒕𝒔),

In illustration,
Checking: -5 0 12

𝑥 ≥ 12, let’s try 𝑥 = 13 𝑥 ≤ −5, let’s try 𝑥 = −5

Substitute from the original Substitute from the original


equation. equation.

𝑥 2 + (𝑥 − 7)2 ≥ 132 𝑥 2 + (𝑥 − 7)2 ≥ 132

(13)2 + (13 − 7)2 ≥ 132 (−5)2 + (−5 − 7)2 ≥ 132

169 + 36 ≥ 169 25 + 144 ≥ 169

205 ≥ 169 179 ≥ 169

TRUE TRUE

This means that the solution set is 𝒙 ≥ 𝟏𝟐, 𝒙 ≤ 𝟓 𝒐𝒓 − 𝟓 ≥ 𝒙 ≥ 𝟏𝟐

Learning Competency with Code:


Solves problems involving quadratic inequalities (M9AL-If-g-1)

Directions: Read, study carefully and understand the following activities. Answer with
honesty.

ACTIVITY 1: Choose The Best!


Directions: How far do you know about the lesson? Choose the letter that you think best
answers the question. For items 3 and 5, solve and show your solution.

For 1-3, The length of a wall is 17 m more than its width. The area of the wall is less than 60𝑚2 .
1. What inequality symbol will be used to represent the given problem?
A. < B. > C. ≤ D. ≥
2. Which of the following is the mathematical sentence for the given problem?
A. 𝑥(𝑥 + 17) < 60𝑚2 C. 𝑥 (𝑥 + 17) > 60𝑚2
2
B. 𝑥(𝑥 + 17) ≤ 60𝑚 D. 𝑥(𝑥 + 17) ≥ 60𝑚2
3. Which of the following could be its length?
A. 3m B. 16m C. 18m D. 20m

59
Note: Practice Personal Hygiene protocols at all times.
For 4-5, The length of the garden is 5m longer than its width and the area is more than 36𝑚2 .
4. Which is the correct mathematical sentence for the problem?
A. 𝑥 (𝑥 + 5) > 36𝑚2 C. 𝑥 (𝑥 + 5) < 36𝑚2
B. 𝑥 (𝑥 + 5) ≥ 36𝑚2 D. 𝑥 (𝑥 + 5) ≤ 36𝑚2
5. Which can be the width of the garden?
A. 2m B. 3m D. 4m D. 5m

Rubrics for Scoring


Numbers 1,2 and 4- 1 point each( 0.5 if incorrect)
For items 3 and 5
4 points-correct solution and correct answer
3 points- correct solution but wrong answer
2 points-wrong solution but correct answer
1 point- wrong solution and wrong answer

ACTIVITY 2: Think Deeply and Solve Me!


Directions: Analyze and solve what is asked in each problem below and use the questions as
your guide.
1. The floor of a conference hall can be covered completely with tiles. Its length is
36ft longer than its width. The area of the floor is less than 2,040 square feet.
a. How would you represent the width of the floor? How about its length?
b. What mathematical sentence would represent the given situation?
c. What are the possible dimensions of the floor? How about the possible areas of
the floor?
2. The area of a rectangle is at most 360 square inches. The length is 6 more than twice
the width. Find the length and width of the rectangle.
a. How would you represent the dimensions of the rectangle?
b. What mathematical sentence would represent the given situation?
c. What are the possible dimensions of the rectangle? Show your solution.

RUBRICS FOR PROBLEM SOLVING


5-the problem is correctly represented, mathematical sentence is accurate and computations are
correct and logically presented.
4- the problem is correctly represented, mathematical sentence is accurate and computations
are correct.
3-the problem is correctly represented, mathematical sentence is accurate and generally, most
of the computations are incorrect.
2-the problem is correctly represented, mathematical sentence is accurate BUT errors in the
computations are severe.
1- the problem is incorrectly represented, mathematical sentence is inaccurate and errors in the
computations are severe.

60
Note: Practice Personal Hygiene protocols at all times.
Reflection:

The lesson has provided you with opportunities to describe quadratic inequalities and
their solution sets using practical situations. Real life problems help you to realize that in each
problem, there is/are solution/s.
Have you ever encountered problem/s that has/have more than one solution/s? If there
is/are, cite some.
___________________________________________________________________________
___________________________________________________________________________
___________________________________________________________________________
___________________________________________________________________________
___________________________________________________________________________

References:
https://www.expii.com/t/word-problems-with-quadratic-inequalities-4542
https://youtu.be/OYajy-IfgAo
Mathematics 9 LM pp. 110-111

61
Note: Practice Personal Hygiene protocols at all times.
ANSWER KEY

PROBLEM SOLVING
Activity 1
1. A
2. A
3. C
4. A
5. D
Activity 2
1. a. 𝑤 = 𝑥, 𝑙 = 𝑥 + 36
b. 𝑥(𝑥 + 36) < 240
c. Possible Answers:
𝑤 = 15𝑓𝑡, 𝑙 = 51𝑓𝑡 𝐴 = 765𝑓𝑡 2
𝑤 = 22𝑓𝑡, 𝑙 = 58𝑓𝑡 𝐴 = 1276𝑓𝑡 2
𝑤 = 30𝑓𝑡, 𝑙 = 66𝑓𝑡 𝐴 = 1980𝑓𝑡 2
2. a. 𝑤 = 𝑥, 𝑙 = 2𝑥 + 6
b. 𝑥 (2𝑥 + 6) ≤ 360
c. 𝑥 ≤ 12, 𝑥 ≥ −15

Prepared by:

MAYLEEN V. YANTO
Writer

62
Note: Practice Personal Hygiene protocols at all times.
MATHEMATICS GRADE 9
Name of Learner: ____________________________ Grade Level: ________________
Section: ___________________________________ Date: _______________________

LEARNING ACTIVITY SHEET


Modelling Real-Life Situations Using Quadratic Functions

Background Information for Learners


Have you ever tried shooting a basketball to the rim? Have you ever wondered why
Stephen Curry, an NBA player is good at shooting the ball? Have you ever experienced
launching an improvised rocket during a Science experiment? Or simply, throwing a stone
upward to hit a mango fruit?
One of the most interesting topics in Mathematics is the quadratic function. It has many
applications and has played a fundamental role in solving many problems related to human life.
This can be helpful in the fields of sports, medicines, business and others.
For example, try to look at the figure below. A basketball player is trying to shoot the
basketball to the rim with a distance of approximately 5 meters. For him to score, he needs to
throw needs to throw the ball vertically upward towards the rim.

5
Height in meters

1
1 2 3 4 5 6

Horizontal distance in meters

The path of the ball is parabolic. It arcs up in the air .The maximum height required for
a perfect shot is 6 meters.
In physics, the distance S (in meters) travelled by a freely falling body after t seconds
can be modelled as S = 4.9t². The two quantities are related quadratically.
Quadratic functions can be also applied in area related problems. For example, the area
A of a rectangular tulip garden has length 5 units longer than thrice its width. This can be
modelled as:

63
Note: Practice Personal Hygiene protocols at all times.
w

3w + 5

Since the area formula is just multiplying the length and the width,
A = w( 3w + 5).
A = 3w² + 5w
Learning Competency with Code: Models real-life situation using quadratic functions
(M9AL-Ig-2)

Directions: Read, study carefully and understand the following activities. Answer with
honesty.

ACTIVITY 1: LAUNCH ATTACK!


Directions: Analyze the problem and answer the given questions.

Horizontal distance in meters

Problem 1. A ball on the football field was kicked by Bruno. The parabolic path of the ball is
traced by the graph above.
Questions 1. How would you describe the graph?___________________________________
2. What is the initial height of the ball?_________________________________
3. What is the maximum height reach by the ball?_________________________
4. Determine the horizontal distance that corresponds to the maximum height
reached by the ball._________________________
5. Approximate the height of the ball it has travelled 3 meters horizontally.______
6. How far does the ball travel horizontally before it hits the ground?_______

Rubrics for Scoring

1-6 -------2 points for every correct answer,


1 point for every incorrect answer

64
Note: Practice Personal Hygiene protocols at all times.
ACTIVITY 2. Sketch me.
Directions: On a Cartesian plane, model the situation below by drawing the parabolic
path. Answer the question that follows.
1. A dolphin jumped off from the water and reached a maximum height of 8 meters above
the sea level with a horizontal distance of 6 meters.

Question. What is the maximum horizontal distance reached by the


dolphin?_______________________

Rubric
10 points for the correct sketch of the parabolic path of the dolphin/
5 points for incorrect sketch
5 points for the correct answer on the question/
2 points for the incorrect answer

ACTIVITY 3. Name it!


Directions: Name 10 objects/situations that are parabolic.
Example: basin
1. 6.
2. 7.
3 8.
4 9.
5 10.

Rubric:
2 points for every correct answer
1 point for every incorrect answer

65
Note: Practice Personal Hygiene protocols at all times.
Reflection
This lesson has provided you with opportunities to model real life situation using
quadratic functions. It helps you realize the importance of Mathematics in human life. Have
you ever tried solving your own problem with the use of Mathematics? If there are/is cite
some:______________________________________________________________________
___________________________________________________________________________
___________________________________________________________________________
___________________________________________________________________________
___________________________________________________________________________
___________________________________________________________________________

References

Mathematics 9 Learner’s Material p.151


http://study.com/academy/practice/quiz-worksheet-modelling-quadratic-functions.html

66
Note: Practice Personal Hygiene protocols at all times.
ANSWER KEY
ACTIVITY 1: Launch Attack!
Directions: Analyze the problem and answer the given questions.

Horizontal distance in meters

Problem 1. A ball on the football field was kicked by Bruno. The parabolic path of the ball is
traced by the graph above.
Questions 1. How would you describe the graph?
Answer: The graph is parabolic.
2. What is the initial height of the ball?
Answer: The initial height of the ball is 0 meters.
3. What is the maximum height reach by the ball?
Answer: The ball reached a maximum height of 6 meters.
4. Determine the horizontal distance that corresponds to the maximum height
reached by the ball._________________________
Answer: The horizontal distance that corresponds to the maximum height
reached by the ball is 4.5 meters
5. Approximate the height of the ball it has travelled 3 meters horizontally.
Answer: The height of the ball is 5 meters.
6. How far does the ball travel horizontally before it hits the ground?
Answer: It has travelled 9 meters.
ACTIVITY 2. Sketch me.
1. A dolphin jumped off from the water and reached a maximum height of 8 meters above
the sea level with a horizontal distance of 6 meters.

Question. What is the maximum horizontal distance reached by the dolphin?


Answer: The dolphin has reached a maximum horizontal distance of 12 meters.
67
Note: Practice Personal Hygiene protocols at all times.
ACTIVITY 3. Name it!
Directions: Name 10 objects/situations that are parabolic.
Example: basin
1. 6.
2. 7
3 8
4 9
5 10

Answer: Students’ answers may vary. The teacher will assess their answers.

Prepared by:

KING RICHER N. TABABA


Writer

68
Note: Practice Personal Hygiene protocols at all times.
MATHEMATICS GRADE 9
Name of Learner: ______________________________ Grade Level: __________
Section: ______________________________________ Date: ________________

LEARNING ACTIVITY SHEET


Representations of Quadratic Functions

Background Information for Learners


A. Representing a Quadratic Function Using Table of Values, Ordered Pairs,
Mapping

Quadratic Function is in the form 𝑦 = 𝑎𝑥 2 + 𝑏𝑥 + 𝑐


where: x values are called DOMAIN
y values are called RANGE
Ordered Pair: (𝑥, 𝑦) 𝑜𝑟 (𝑑𝑜𝑚𝑎𝑖𝑛, 𝑟𝑎𝑛𝑔𝑒)

Note: A quadratic relation is NOT a quadratic function if the domain is repeated,


then it is considered a PURE RELATION.

Examples:
a) Set of ordered pairs
(3, 2), (2, −3), (1, 0), (4, 2)
domain → 3, 2, 1, 4, → NOT REPEATED
range → 2, −3, 0, −2 → NOT REPEATED
It is a one-to-one correspondence, therefore the set of ordered pairs is a
FUNCTION.

b) Table of Values
X -3 -1 1 3
Y 5 0 5 3

domain → −3, −2, 1, 3 → NOT REPEATED


range → 5, 0, 5, 3 → REPEATED
It is many-to-one correspondence, therefore the table of values is a FUNCTION.

c) Mapping
X y domain → 2, 2, 3, 4 → REPEATED
2 -1
3 -2 range → −1, − 2, −3, −4 → NOT REPEATED
4 -3
-4 It is a one-to-many correspondence, therefore the given

mapping is PURE RELATON.


69
Note: Practice Personal Hygiene protocols at all times.
B. Identifying Whether a Given Equation is a Quadratic Function

In the previous lesson of Quadratic Equation, you were informed how to identify an
equation in the form of 𝑎𝑥 2 + 𝑏𝑥 + 𝑐=0 This topic aims to broaden your knowledge to
quadratic function in the form 𝑦 = 𝑎𝑥 2 + 𝑏𝑥 + 𝑐 where a,b, and c are real numbers and a is
not equal to zero.
Consider that x is in the second degree power but not y.
Examples:
1. 𝑦 = 2𝑥 2 − 𝑥 + 2 → QUADRATIC FUNCTION
2. 𝑦 = 𝑥(𝑥 − 3), simplify, the result is 𝑦 = 𝑥 2 − 3𝑥 → QUADRATIC FUNCTION
3. 𝑦 2 + 𝑥 2 = 3 , y is in the second degree → NOT QUADRATIC FUNCTION

C. Identifying Quadratic Functions Given Graphs

Vertical Line Test- a graph is a function if and only if a vertical line touches the graph only
ONCE.

Examples:

1. Vertical Line
Once→ 𝑭𝑼𝑵𝑪𝑻𝑰𝑶𝑵

2. More than Once → 𝑷𝑼𝑹𝑬 𝑹𝑬𝑳𝑨𝑻𝑰𝑶𝑵

Learning Competency with Code:


Represents a quadratic function using a) table of values, b) graph, c) equation (M9AL-
Ig-3)

Directions: Read, study carefully and understand the following activities. Answer with
honesty.

70
Note: Practice Personal Hygiene protocols at all times.
ACTIVITY 1: Work on this!
Directions/Instruction: Give the correspondence and tell whether the given is a FUNCTION or
PURE RELATION.
CORRESPONDENCE ANSWER

1. (5, −1)(−1, 3)(−2, −1)(3, 5) _____________________ _______________

2. (0, 3)(5, 0)(3, −3)(−3, −5) _____________________ _______________

3. (1, 2)(−2, −1)(2, −2)(−1, 0) ____________________ _______________

4.
x 1 -7 -1 -5
y 7 -1 5 -7 _____________________ _______________

5.

x 4 -1 -2 4 2 _____________________ _______________
y 3 1 -1 -4 -2

6.
x 5 2 -2 -5 3 ____________________ _______________
y 3 -4 3 5 4

7.
___________________ ____________

8. ___________________ _______________

9.
___________________ _______________

10.

___________________ _______________

71
Note: Practice Personal Hygiene protocols at all times.
Activity 2: Identify Me!
Directions: Identify which of the following equations is a Quadratic Function or NOT a
Quadratic Function. Simplify in the form 𝑦 = 𝑎𝑥 2 + 𝑏𝑥 + 𝑐 if necessary. Write QF if it is a
Quadratic Function and NQF if it is NOT a Quadratic Function.
______________1. 𝑦 = 𝑥 2 − 5𝑥 + 1
______________2. 3𝑥 + 𝑦 − 3 = 0
______________3. 𝑦(𝑦) = 3𝑥 + 𝑦 + 1
______________4. 𝑦 + 𝑥 2 = 2𝑥 2 − 4𝑥 − 1
______________5. 𝑦 − 2 = 𝑥 2 − 2
Note: For numbers 6-10, simplify each given into 𝒚 = 𝒂𝒙𝟐 + 𝒃𝒙 + 𝒄
_______________6. 𝑦 = 𝑥 (𝑥 + 2) − 5
_______________7. 𝑦(2𝑦 + 1) = 3𝑥 2 + 2𝑦 2 + 4𝑥
_______________8. 𝑦 = (𝑥 + 1)(𝑥 − 1)
_______________9. 𝑦 = 3𝑥 + 2𝑥 − 3
_______________10. 𝑦 = (2𝑥 + 1)2 − 4𝑥 2 − 5
Rubrics for Scoring
1-5→ 1 point each 0.5 each for incorrect answer
6-10→ 2 points each 1 point each for incorrect answer
Activity 3: Graph Analysis
Directions: Use vertical line test to determine if the given graph is a FUNCTION or PURE
RELATION.

1. 2. 3.

Answer: _____________ Answer: _______________ Answer: _____________

4. 5. 6.

Answer: _____________ Answer: ________________ Answer: _____________

7. 8. 9.

Answer: _______________ Answer: _______________ Answer: ______________


72
Note: Practice Personal Hygiene protocols at all times.
10.

Answer: __________________

Guide Questions:
1. Which graphs are quadratic? Explain your answer.
2. Which graphs are not quadratic? Explain your answer.

Rubrics for Scoring


1-10→ 2 𝑝𝑜𝑖𝑛𝑡𝑠 𝑒𝑎𝑐ℎ (1 point each if incorrect)

Reflection
One-to-one, many-to-one correspondence work as function but NEVER to one-
to-many. How do you relate this to real-life situation?
_____________________________________________________________________
_____________________________________________________________________
_____________________________________________________________________
_____________________________________________________________________
_____________________________________________________________________

Reference
Mathematics Learner’s Material 9 First Edition, 2004 Department of Education, pp. 147-150

73
Note: Practice Personal Hygiene protocols at all times.
ANSWER KEY
ACTIVITY 1: Work on this!
CORRESPONDENCE ANSWER

1. (5, −1)(−1, 3)(−2, −1)(3, 5) many-to-one Function

2. (0, 3)(5, 0)(3, −3)(−3, −5) one -to-one Function

3. (1, 2)(−2, −1)(2, −2)(−1, 0) one -to-one Function

4. one -to-one Function


x 1 -7 -1 -5
y 7 -1 5 -7

5.
X 4 -1 -2 4 2
one -to-many Pure Relation
Y 3 1 -1 -4 -2
6.
x 5 2 -2 -5 3
many-to-one Function
Y 3 -4 3 5 4

7.
one -to-one Function

8.
one -to-many Pure Relation

9.
many-to-one Function

10. one-to-many Pure Relation

74
Note: Practice Personal Hygiene protocols at all times.
Activity 2: Identify Me
1. QF
2. NQF
3. NQF
4. QF
5. QF
6. QF 𝑦 = 𝑥 2 + 2𝑥 − 5
7. QF 𝑦 = 3𝑥 2 + 4𝑥
8. QF 𝑦 = 𝑥 2 − 1
9. NQF
10. NQF

Activity 3: Graph Analysis


1. Function 6. Pure Relation
2. Function 7. Function
3. Pure Relation 8. Function
4. Function 9. Function
5. Pure Relation 10. Function

Prepared by:

AGNES M. TALDE
Writer

75
Note: Practice Personal Hygiene protocols at all times.
MATHEMATICS GRADE 9
Name of Learner: ________________________________ Grade Level: ____________
Section: ____________________________________ Date: __________________

LEARNING ACTIVITY SHEET

Transforming Quadratic Functions From General Form To Vertex Form


And Vice Versa

Background Information for Learners

A. Transforming General Form of Quadratic Function 𝒚 = 𝒂𝒙𝟐 + 𝒃𝒙 + 𝒄 into


Vertex Form 𝒚 = 𝒂(𝒙 − 𝒉)𝟐 + 𝒌
Quadratic Function requires vertex and shall be considered as point on its graph. The
vertex form 𝑦 = 𝑎(𝑥 − ℎ)2 + 𝑘 will help you work on problems involving the vertex of the
graph of a quadratic function.
Vertex Form: 𝑦 = 𝑎(𝑥 − ℎ)2 + 𝑘, where: k is the range,
a is the coefficient of quadratic term,
h is the line/axis of symmetry
Note: (x,y)→ ordered pair or a point
(h,k)→vertex of the graph
𝒃 4𝑎𝑐−𝑏2
Consider these in solving for h and k: ℎ = − 𝟐𝒂 and 𝑘= 4𝑎
Examples:
1. 𝑦 = 𝑥 2 − 2𝑥 + 3 𝑎 = 1, 𝑏 = −2, 𝑐 = 3
Solve for h and k:
𝒃 −2 −2
ℎ = − 𝟐𝒂 Substitute: ℎ = − [2(1)] = − ( 2 ) =1, 𝒉=𝟏
4𝑎𝑐−𝑏2 4(1)(3)−(−2)2 12−4 8
𝑘= Substitute: 𝑘 = = =4 , 𝒌=𝟐
4𝑎 4(1) 4
Vertex: (1,2)

Substitute to 𝑦 = 𝑎(𝑥 − ℎ)2 + 𝑘


𝑦 = (𝑥 − 1)2 + 2
Note: a=1, so there is no need to write it in the vertex form.
Therefore, the general form 𝒚 = 𝒙𝟐 − 𝟐𝒙 + 𝟑 has vertex form 𝒚 = (𝒙 − 𝟏)𝟐 + 𝟐

2. 𝑦 = −3𝑥 2 + 𝑥 − 5𝑎
a= -3, b=1, c= -5
Solve for h and k
𝒃 1 1 𝟏
ℎ= =− = −( ) 𝒉=
𝟐𝒂 2(−3) −6 𝟔

76
Note: Practice Personal Hygiene protocols at all times.
4𝑎𝑐−𝑏2 4(−3)(−5)−(1)2 60−1 59 𝟓𝟗
𝑘= 4𝑎
= 4(−3)
= −12
= −12 𝒌 = − 𝟏𝟐
𝟏 𝟓𝟗
Vertex: (𝟔 , − 𝟏𝟐)
𝟏 𝟓𝟗
The general form 𝒚 = −𝟑𝒙𝟐 + 𝒙 − 𝟓 has the vertex form 𝒚 = −𝟑(𝒙 − 𝟔)𝟐 − 𝟏𝟐

B. Transforming Vertex Form 𝒚 = 𝒂(𝒙 − 𝒉)𝟐 + 𝒌 Into General Form 𝒚 = 𝒂𝒙𝟐 + 𝒃𝒙 + 𝒄

You have learned how to express or transform general form 𝑦 = 𝑎𝑥 2 + 𝑏𝑥 + 𝑐 into vertex
form 𝑦 = 𝑎(𝑥 − ℎ)2 + 𝑘. The previous lesson gave you knowledge that (h,k) is the vertex of
the graph.
In this lesson, we will do the reverse order by transforming vertex form into general
form. Before going further, we will have a simple review on how to square binomials.

Rule: (𝑎 + 𝑏)2 → square the first term→ 𝑎2


→twice the product of the first and the second term a and b→ 2𝑎𝑏
→ square the second term→ 𝑏2
Therefore: (𝒂 + 𝒃)𝟐 = 𝒂𝟐 + 𝟐𝒂𝒃 + 𝒃𝟐

Transform Vertex form into General form or 𝑦 = 𝑎(𝑥 − ℎ)2 + 𝑘 into 𝑦 = 𝑎𝑥 2 + 𝑏𝑥 + 𝑐

Examples: Transform into general form 𝑦 = 𝑎𝑥 2 + 𝑏𝑥 + 𝑐


a. 𝑦 = 3(𝑥 − 2)2 + 5, first simplify (𝑥 − 2)2 𝑥 2 − 4𝑥 + 4,
then multiply the result by 3 𝑦 = 3(𝑥 2 − 4𝑥 + 4) + 5
𝑦 = 3𝑥 2 − 12𝑥 + 12 + 5
Combine Like Terms 𝑦 = 3𝑥 2 − 12𝑥 + 17
General Form: 𝒚 = 𝟑𝒙𝟐 − 𝟏𝟐𝒙 + 𝟏𝟕

b. 𝑦 = −5(𝑥 + 3)2 − 1 First, simplify (𝑥 + 3)2 𝑥 2 + 6𝑥 + 9


next, multiply the result by 5 5(𝑥 2 + 6𝑥 + 9) − 1 5𝑥 2 + 30𝑥 + 45 − 1
Combine like terms 𝑦 = −5𝑥 2 − 30𝑥 − 46
General Form: 𝒚 = −𝟓𝒙𝟐 − 𝟑𝟎𝒙 − 𝟒𝟔

Learning Competency with Code: Transforms the quadratic function defined by y= ax2 + bx
+ c into the form y = a(x – h)2 +k (M9AL-Ih-1)

Directions: Read, study carefully and understand the following activities. Answer with
honesty.

77
Note: Practice Personal Hygiene protocols at all times.
ACTIVITY 1: What’s My Vertex?

Directions: The following are expressed in vertex forms, determine the vertex of each item.

A. Given Vertex
1. 𝑦 = (𝑥 + 3)2 + 4 ___________________
2. 𝑦 = (𝑥 − 8)2 + 5 ___________________
3. 𝑦 = −(𝑥 + 6)2 − 2 ___________________
4. 𝑦 = 3(𝑥 − 1)2 − 2 ____________________
5. 𝑦 = −3(𝑥 + 2)2 + 1 ____________________
6. 𝑦 = 4(𝑥 + 5)2 − 3 ____________________
1
7. 𝑦 = −5(𝑥 − 7)2 + 4 ____________________
1 1
8. 𝑦 = (𝑥 − 2)2 + 3 ____________________
3
9. 𝑦 = −(𝑥 + 5)2 + 2 ____________________
2 3
10. 𝑦 = −2(𝑥 + 3)2 − 5 ____________________

Rubrics for Scoring


2 points- if both h and k are correct
1 point-h is incorrect and k is correct or vice versa
0.5 point-if both h and k are incorrect

78
Note: Practice Personal Hygiene protocols at all times.
ACTIVITY 2: Perfect Match!
It is a small European country with the flag given below, what country is it?

Directions: To find its answer, transform the following quadratic functions in vertex form.
Write the letter that corresponds to each answer in the boxes below. Show your solution.

____________1. 𝑦 = 𝑥 2 + 4𝑥 − 5 R 1 25
𝑦 = 4(𝑥 − )2 −
____________2. 𝑦 = −𝑥 2 + 6𝑥 − 2 4 4
2
____________3. 𝑦 = 𝑥 2 − 8𝑥 + 10 X 𝑦 = (𝑥 − 4) − 6
____________4. 𝑦 = −𝑥 2 − 2𝑥 + 11 E 𝑦 = −(𝑥 + 1)2 + 10
____________5. 𝑦 = −2𝑥 2 + 𝑥 − 3 O 𝑦 = 2(𝑥 + 1) − 8
____________6. 𝑦 = 3𝑥 2 + 2𝑥 − 5 U 𝑦 = −(𝑥 − 3)2 − 11
____________7. 𝑦 = 2𝑥 2 + 4𝑥 − 6 1 16
G
____________8. 𝑦 = −5𝑥 2 + 2𝑥 − 4 𝑦 = −3(𝑥 + )2 +
3 3
____________9. 𝑦 = 4𝑥 2 − 2𝑥 − 6 1 23
M
____________10. 𝑦 = −3𝑥 2 − 2𝑥 + 5 𝑦 = −2(𝑥 − )2 −
4 8
2
L 𝑦 = (𝑥 + 2) − 9
B 1 16
𝑦 = 3(𝑥 + )2 −
3 3
U 2
1 19
𝑦 = −5 (𝑥 + ) −
5 5

ANSWER: ___ ____ ___ ____ ____ ____ ____ ____ ____ ____

RUBRICS:
2-the computations are correct
1-generally, most of the computations are not correct.

ACTIVITY 3: Transformers!

A. Solve for the vertex (h, k) of the following quadratic functions.


1. 𝑦 = 𝑥 2 + 7𝑥 − 3 _____________________
2. 𝑦 = −𝑥 2 − 5𝑥 − 12 ______________________
2
3. 𝑦 = −3𝑥 + 𝑥 − 2 ______________________
2
4. 𝑦 = 2𝑥 − 2𝑥 + 1 ______________________
2
5. 𝑦 = 2𝑥 − 4𝑥 − 3 ______________________

79
Note: Practice Personal Hygiene protocols at all times.
B. Solve for h and k in each item and transform into vertex form 𝑦 = 𝑎(𝑥 − ℎ)2 + 𝑘
Vertex Vertex Form
6. 𝑦 = 2𝑥 2 − 𝑥 + 3 _______________ ___________________________
7. 𝑦 = −𝑥 2 + 2𝑥 + 5 _______________ ___________________________
8. 𝑦 = 4𝑥 2 − 3𝑥 + 1 _______________ ___________________________
2
9. 𝑦 = 5𝑥 − 3𝑥 + 2 _______________ ___________________________
10. 𝑦 = −3𝑥 2 − 𝑥 + 4 _______________ ___________________________

Rubrics for Scoring


3- all computations are correct
2-generally most of the computations are not correct
1-errors in computations are severe

ACTIVITY 4: Gods & Goddesses


Directions: Identify the Gods/Goddesses illustrated below by matching the vertex form to its
correct general form. Write the God/ Goddesses before each number.
_____________________1. 𝑦 = (𝑥 − 2)2 + 1 God of Strength
2
_____________________2. 𝑦 = (𝑥 − 8) + 6 God of darkness
2
_____________________3. 𝑦 = (𝑥 − 2) − 2 God of Time
2
_____________________4.𝑦 = −3(𝑥 − 8) + 5 God of love & procreation
2
______________________5. 𝑦 = −2(𝑥 + 3) + 6 God of the Dead
______________________6. 𝑦 = 3(𝑥 + 3)2 − 9 God of Dreams and Sleep
_____________________7. 𝑦 = 4(𝑥 − 7)2 + 4 Goddess of Love and Beauty
2
_____________________8. 𝑦 = −8(𝑥 + 5) + 1 Goddess of Wisdom, Poetry & Art
_____________________9. 𝑦 = −2(𝑥 − 5)2 − 2 Goddess of Night
____________________10. 𝑦 = −5(𝑥 − 2)2 + 7 Goddess of fame and Gossip

ATHENA 𝑦 = −8𝑥 2 − 80𝑥 − 199


HADES 𝑦 = −2𝑥 2 − 12𝑥 − 12

KRATOS 𝑦 = 𝑥 2 − 4𝑥 + 5 NYX 𝑦 = −2𝑥 2 + 20𝑥 − 52

CHRONOS 𝑦 = 𝑥 2 − 4𝑥 + 2 MORPHEUS 𝑦 = 3𝑥 2 + 18𝑥 + 18

EROS 𝑦 = −3𝑥 2 + 48𝑥 − 187 PHEME 𝑦 = −5𝑥 2 + 20𝑥 − 13

APHRODITE 𝑦 = 4𝑥 2 − 56𝑥 + 200 EREBUS 𝑦 = 𝑥 2 − 16𝑥 + 70

Rubrics for Scoring


2 points-if the answer is correct
1 point- if the answer is incorrect

80
Note: Practice Personal Hygiene protocols at all times.
ACTIVITY5: Write It In The Form…
Directions: Transform 𝑦 = 𝑎(𝑥 − ℎ)2 + 𝑘 into general form 𝑦 = 𝑎𝑥 2 + 𝑏𝑥 + 𝑐. Show your
complete solution.
Vertex Form: 𝒚 = 𝒂(𝒙 − 𝒉)𝟐 + 𝒌 General Form: 𝒚 = 𝒂𝒙𝟐 + 𝒃𝒙 + 𝒄
1. 𝑦 = (𝑥 − 3)2 + 2 ______________________________
2
2. 𝑦 = −(𝑥 − 5) + 3 ______________________________
3. 𝑦 = 2(𝑥 − 1)2 + 4 ______________________________
( ) 2
4. 𝑦 = −3 𝑥 + 5 − 1 ______________________________
5. 𝑦 = 5(𝑥 − 2)2 + 10 ______________________________
2
6. 𝑦 = (𝑥 + 7) − 5 ______________________________
7. 𝑦 = −2(𝑥+3)2 + 9 ______________________________
2
8. 𝑦 = −(𝑥 + 1) − 6 ______________________________
9. 𝑦 = 4(𝑥 − 6)2 + 10 ______________________________
2
10. 𝑦 = 3(𝑥 + 4) − 7 ______________________________

Rubrics for Scoring


4 points- if all the terms are correct
3 points- if 2 terms are correct, 1 term is incorrect
2 points- if 1 term is correct, 2 terms are incorrect
1 point- if all terms are incorrect

Reflection
This lesson strengthens one’s ability on the laws of integers. Sometimes, if we
transform one person, he can be better or worse. Which transformation do you prefer? Why?
___________________________________________________________________________
___________________________________________________________________________
___________________________________________________________________________
___________________________________________________________________________
___________________________________________________________________________

Reference Mathematics 9 Learner’s Material pp. 129-135

81
Note: Practice Personal Hygiene protocols at all times.
ANSWER KEY
ACTIVITY 1: What’s My Vertex?
1. (−3,4) 6. (−5, −3)
2. (8,5) 7. (7,1/4)
1 1
3. (−6, −2) 8. (2 , 3)
3
4. (1, −2) 9. (− 5 , 2)
2 3
5. (−2,1) 10. (− 3 , − 5)

ACTIVITY 2: Perfect Match!


LUXEMBOURG
ACTIVITY 3: Transformers!
7 61 1 1
A. 1. (− , − ) 4. ( , )
2 4 2 2
5 23
2. (− , − ) 5. (1, −5)
2 4
1 23
3. ( , − )
6 12
B. VERTEX VERTEX FORM
1 23 1 23
6. ( , ) 𝑦 = 2(𝑥 − )2 +
4 8 4 8
7. (1,6) 𝑦 = −(𝑥 − 1)2 + 6
3 7 3 7
8. ( , ) 𝑦 = 4(𝑥 − )2 +
8 16 8 16
3 31 3 2 21
9. ( , ) 𝑦= 5(𝑥 − ) +
10 20 10 20
1 49 1 2 49
10(− , ) 𝑦= −3(𝑥 + ) +
6 12 6 12
ACTIVITY 4: Gods & Goddesses
1. KRATOS 6. MORPHEUS
2. EREBUS 7. APHRODITE
3. CHRONOS 8. ATHENA
4. EROS 9. NYX
5. HADES 10. PHEME
ACTIVITY 5: Write It In The Form…
1. 𝑦 = 𝑥 2 − 6𝑥 + 11
2. 𝑦 = −𝑥 2 + 10𝑥 − 22
3. 𝑦 = 2𝑥 2 − 4𝑥 + 6
4. 𝑦 = −3𝑥 2 − 30𝑥 − 76
5. 𝑦 = 5𝑥 2 − 20𝑥 + 30
6. 𝑦 = 𝑥 2 + 14𝑥 + 44
7. 𝑦 = −2𝑥 2 − 12𝑥 − 9
8. 𝑦 = −𝑥 2 − 2𝑥 − 7
9. 𝑦 = 4𝑥 2 − 48𝑥 + 154
10. 𝑦 = 3𝑥 2 + 24𝑥 + 41
Prepared by:
MAYLEEN V. YANTO
Writer
82
Note: Practice Personal Hygiene protocols at all times.
MATHEMATICS GRADE 9
Name of Learner: ______________________________ Grade Level: __________
Section: ______________________________________ Date: ________________

LEARNING ACTIVITY SHEET


Graphs of Quadratic Functions

Background Information for Learners

A. Completing The Table of Values, Solving For The Range Given The Domain,
And Determining The Graph And Intercepts

Given a quadratic function in the form 𝑦 = 𝑎𝑥 2 + 𝑏𝑥 + 𝑐, you are tasked to


complete the table of values and sketch the graph on the Cartesian plane.
On the graph, plot the ordered pairs (x, y) where x values are called DOMAIN
and y values are called RANGE.
The example below will guide you on this activity.
a) Complete the table and sketch on the Cartesian plane, use 𝑦 = 𝑥 2 − 2𝑥 − 3, substitute
the values of x in the given function.

X -3 -2 -1 0 1 2 3
Y 12 5 0 -3 -4 -3 0

when 𝑥 = −3, 𝑦 = (−3)2 − 2(−3) − 3, 𝑦 = 9 + 6 − 3, 𝒚 = 𝟏𝟐


𝑥 = −2, 𝑦 = (−2)2 − 2(−2) − 3, 𝑦 = 4 + 4 − 3, 𝒚=𝟓
𝑥 = −1, 𝑦 = (−1)2 − 2(−1) − 3, 𝑦 = 1 + 2 − 3, 𝒚=𝟎
𝑥 = 0, 𝑦 = (0)2 − 2(0) − 3, 𝑦 = 0 − 3, 𝒚 = −𝟑
𝑥 = 1, 𝑦 = (1)2 − 2(1) − 3, 𝑦 = 1 − 2 − 3, 𝒚 = −𝟒
𝑥 = 2, 𝑦 = (2)2 − 2(2) − 3, 𝑦 = 4 − 4 − 3, 𝒚 = −𝟑
𝑥 = 3, 𝑦 = (3)2 − 2(3) − 3, 𝑦 = 9 − 6 − 3, 𝒚=𝟎
Domain are: -3, -2, -1, 0, 1, 2, 3
Range are: 12, 5, 0, -3, -4, -3, 0
Ordered pairs/Points on 𝑦 = 𝑥 2 − 2𝑥 − 3 are:
(-3, 12), (-2, 5), (-1, 0), (0, -3), (1, -4), (2, -3), (3, 0)

83
Note: Practice Personal Hygiene protocols at all times.
Then plot the points on the Cartesian Plane and sketch the graph

Note: Connect the points


and form a CURVE.

Intercepts: x-intercept → when y=0


y-intercept → when x=0
Observe the ordered pairs table of values, there exist zero values in the range
and domain, which means that the given function 𝑦 = 𝑥 2 − 2𝑥 − 3 has x and y-
intercepts.
Therefore: x-intercepts of the function are -1 and 3
y- intercept is -3

B. Identifying the Parts and Properties of the Graph of Quadratic Function

In the previous lessons, you were informed about the vertex (of the graph) which is a
point on the graph.
In this topic, you will be more equipped of the knowledge of the properties of the graph
of quadratic function 𝑦 = 𝑎𝑥 2 + 𝑏𝑥 + 𝑐 called the PARABOLA.

Parts and Properties of Parabola


1. Axis/ line of Symmetry- a line that divides a parabola into 2 equal parts such that one-
half of the graph is a reflection of the other half. The line x = h is the axis of symmetry.
2. Vertex- is either the lowest or the highest point of the graph.

Remember this:
a) If 𝑎 > 0, the parabola opens upward and has a minimum point.
b) If 𝑎 < 0, the parabola opens downward and has a maximum point.

Vertex of the graph is defined as (h, k) refer this to the previous lesson.

84
Note: Practice Personal Hygiene protocols at all times.
3. Range- is the set of {𝑦: 𝑦 ≥ 𝑘}
Remember this:
If the opening is upward, then the range is 𝑦 ≥ 𝑘 and if downward, the range is 𝑦 ≤ 𝑘.

The graphs below show clearly the parts and properties of a parabola.

Summary:
1. In g1, the following are its parts:
a) opening: downward
b) vertex: (-3, 3) h= -3, k=3
c) axis of symmetry : x= -3
d) point of the vertex: maximum point
e) range : y≤3
2. In g2, the following are its parts:
a) opening : upward
b) vertex : (4, -2) h= 4, k= -2
c) axis of symmetry : x= 4
d) point of the vertex: minimum point
e) range : y ≥ −2

Note: Axis of symmetry is expressed in x=h

C. Plotting the Graph of Quadratic Function in the Form 𝒚 = 𝒂𝒙𝟐 + 𝒃𝒙 + 𝒄 and


Identifying its Parts

This topic has something to do with the previous lesson; the vertex of the
𝑏 4𝑎𝑐−𝑏2
parabola ℎ = − 2𝑎 , 𝑘 = 4𝑎
Suppose you are tasked to plot the graph of 𝑦 = 𝑎𝑥 2 + 𝑏𝑥 + 𝑐 , notice
that a is either positive or negative, so you will consider the opening( upward 𝑎 > 0, and
downward 𝑎 < 0)
Examples:
a) 𝑦 = 𝑥 2 − 2𝑥 − 3 Solve the vertex (h, k)
𝑎 = 1, 𝑏 = −2, 𝑐 = −3
85
Note: Practice Personal Hygiene protocols at all times.
𝑏 −2 −2
a.1. ℎ = − , ℎ = −[ ] , ℎ = − ( ) , ℎ = −(−1), ℎ = 1
2𝑎 2(1) 2
4𝑎𝑐−𝑏2 4(1)(3)−(−2)2 −12−4 −16
𝑘= ,𝑘 = = = , 𝑘 = −4
4𝑎 4(1) 4 4
Vertex (1, -4)
a.2. Get a point, preferably y-intercept, where x=0, (0, y) on 𝑦 = 𝑥 2 − 2𝑥 − 3
Point (0, 3) 𝑦 = 𝑥 2 − 2𝑥 − 3
𝑦 = 02 − 2(0) − 3
𝑦 = 0−0−3
𝑦 = −3
a.3. Draw a Cartesian Plane
Plot Vertex (1, -4), Point or P1 (0, -3)
Connect the points forming a curve.
Note: The distance of the axis of symmetry
to point (0, -3) is 4 units.

Then, draw a reflection of the curve at the right by


counting 4 units from the axis of symmetry, thus,
creating second point or P2 (5, 0)

Parts of the Parabola:


• Opening is upward, 𝑎 > 0
• Vertex→ (1, −4)
• Axis of symmetry→ 𝑥 = 1
• Point of the Vertex →minimum point
• Range → 𝑦 ≥ −4

b) 𝑦 = −𝑥 2 + 4𝑥 − 1
b.1 𝑎 = −1, 𝑏 = 4, 𝑐 = −1
2 2
ℎ = −[ ] , ℎ = − ( ) , ℎ = −(−2), ℎ = 2
2(−1) −2
4(−1)(−1)−(4)2 4−16 −12
𝑘= = = , 𝑘=3
4(−1) −4 −4
Vertex (2, 3)

b. 2 Point or P, x=0 in 𝑦 = −𝑥 2 + 4𝑥 − 1
P1 ↔(0, -1) 𝑦 = −𝑥 2 + 4𝑥 − 1
𝑦 = −(0)2 + 4(0) − 1
𝑦 = −1

86
Note: Practice Personal Hygiene protocols at all times.
b. 3. Draw the Cartesian Plane Plot Vertex (2, 3), Point or P1 (0, -1)

Connect the vertex and P1 by forming a curve.


Count 2 units to the right from the axis of
symmetry to get the second point or P2 (4, -1)

And connect the vertex and P2, then complete the


parabola.

Parts of the Parabola:


• Opening is downward, 𝑎 < 0
• Vertex→ (2, 3)
• Axis of symmetry→ 𝑥 = 2
• Point of the Vertex →maximum point
• Range → 𝑦 ≤ 3

c) 𝑦 = 𝑎(𝑥 − ℎ)2 + 𝑘 → in vertex form


2
c. 1. 𝑦 = 2(𝑥 − 1) − 3
Vertex (1, -3)
Point 1 or P1 (0, -1)

Point 2 or P2 (2, -2)


𝑦 = 2(𝑥 − 1)2 − 3
=2(−1)2 − 3
=2(1) − 3
=2 − 3
𝑦 = −1

Learning Competency with Code:


Graphs a quadratic function (a) domain (b) range (c) intercepts (d) axis of symmetry
(e) vertex (f) direction of the opening of the parabola (M9AL-Ig-h-i-1)

Directions: Read, study carefully and understand the following activities. Answer with
honesty.

87
Note: Practice Personal Hygiene protocols at all times.
ACTIVITY 1: You Completed My Day! To the LEFT, to the RIGHT! Put me UP, put me
DOWN
Directions: Complete the table of values given the functions, plot the points, sketch the graph
and determine the intercepts. Show your complete solutions.

Use this table to solve for y- values (range)


1. 𝑦 = 𝑥 2 − 𝑥 − 6
X -3 -2 -1 0 1 2 3
Y

x-intercepts: ________________________
y-intercepts: ________________________

2. 𝑦 = −𝑥 2 + 4𝑥 − 1
X -3 -2 -1 0 1 2 3
Y

x-intercepts: ________________________
y-intercepts: ________________________

Rubrics for Scoring


2 points each value of range
5 points → accuracy of the graph 2 points for incorrect answer
2 points each value for intercepts

88
Note: Practice Personal Hygiene protocols at all times.
1 point each for incorrect answer
ACTIVITY 2: What are the parts of my body?
Directions: Identify the parts of the following graphs of quadratic functions. Write your
answers on the box.

Graph Opening of Vertex Axis of Point of Range


the Parabola Symmetry Vertex
A 1. 2. 3. 4. 5.

B 6. 7. 8. 9. 10.

C 11. 12. 13. 14. 15.

D 16. 17. 18. 19. 20.

Rubric for Scoring


1 point each blank

89
Note: Practice Personal Hygiene protocols at all times.
ACTIVITY 3: Draw and Describe Me!
Directions: Plot the graph of each quadratic function and identify its parts.

1. 𝑦 = 2𝑥 2 + 4𝑥 − 3
Parts of the Parabola:

a) Vertex _____________________

b) Opening _____________________

c) Axis of Symmetry____________________

d) Point of the Vertex____________________

e) Range _____________________

2. 𝑦 = −𝑥 2 − 2𝑥 −3
Parts of the Parabola:

a) Vertex _____________________

b) Opening _____________________

c) Axis of Symmetry____________________

d) Point of the Vertex____________________

e) Range _____________________

3. 𝑦 = 2(𝑥 − 1)2 − 1
Parts of the Parabola:

a) Vertex _____________________

b) Opening _____________________

c) Axis of Symmetry____________________

d) Point of the Vertex____________________

e) Range _____________________

90
Note: Practice Personal Hygiene protocols at all times.
Rubrics for Scoring

2 points each for the parts, 1 point each or incorrect answer


5 points for the graph, 2 points each for incorrect but with illustration

Reflection
Range depends on domain, one does NOT exist without the other. This topic reminds
you of the saying “NO MAN IS AN ISLAND”, that life is full of joy with the presence of
others. Does this apply in this activity? If YES/NO, why?
___________________________________________________________________________
___________________________________________________________________________
___________________________________________________________________________
___________________________________________________________________________
___________________________________________________________________________
Reference
Mathematics Learner’s Material 9 First Edition, 2004 Department of Education, pp. 151-175

ANSWER KEY
ACTIVITY 1: You Completed My Day! To the LEFT, to the RIGHT! Put me UP, put me
DOWN

1. 𝑦 = 𝑥 2 − 𝑥 − 6
x -3 -2 -1 0 1 2 3
y 6 0 -4 -6 -6 -4 0

x-intercepts: -2 and 3
y-intercepts: -6

91
Note: Practice Personal Hygiene protocols at all times.
2. 𝑦 = −𝑥 2 + 4𝑥 − 1
x -3 -2 -1 0 1 2 3
y -22 -13 -6 -1 2 3 2

x-intercepts: none
y-intercepts: -1

ACTIVITY 2: What are the parts of my body?

Graph Opening of the Vertex Axis of Point of Range


Parabola Symmetry Vertex
A 1. upward 2. (-5, 0) 3. x= -5 4. minimum 5. 𝑦 ≥ 0
point

B 6. downward 7. (4, 2) 8. x= 4 9. maximum 10. 𝑦 ≤ 2


point

C 11. downward 12. (-8, -1) 13. x= -8 14. maximum 15. 𝑦 ≤ −1


point

D 16. upward 17. (5, -3) 18. x= 5 19. minimum 20. 𝑦 ≥ −3


point

ACTIVITY 3: Draw and Describe Me!


1. 𝑦 = 2𝑥 2 + 4𝑥 − 3
Parts of the Parabola:
a) Vertex _______(-1, -5)________
b) Opening ______upward ______
c) Axis of Symmetry_____x= -1 ______
d) Point of the Vertex____minimum point___
e) Range ______𝑦 ≥ −5________

Points on the graph

92
Note: Practice Personal Hygiene protocols at all times.
P1 (0, -3) → y-intercept
P2 (-2, -3)

2. 𝑦 = −𝑥 2 − 2𝑥 − 3

Parts of the Parabola:


a) Vertex _______(-1, -2)________
b) Opening _______downward __
c) Axis of Symmetry_____x= -1 ______
d) Point of the Vertex____maximum point___
e) Range ______𝑦 ≤ −2________

Points on the graph


P1 (0, -3) → y-intercept
P2 (-2, -3)

3. 𝑦 = 2(𝑥 − 1)2 − 1 Parts of the Parabola:


a) Vertex _______(1, -1)________
b) Opening _______upward ____
c) Axis of Symmetry_____x= 1 ______
d) Point of the Vertex____minimum point___
e) Range ______𝑦 ≥ −1________

Points on the graph


P1 (0, 1) → y-intercept
P2 (0, 1)
Prepared by:
AGNES M. TALDE
Writer

93
Note: Practice Personal Hygiene protocols at all times.
MATHEMATICS GRADE 9
Name: Grade Level:
Date: Score:

LEARNING ACTIVITY SHEET


Analyzing Graphs of Quadratic Function

Background Information for Learners


In the previous activities familiarized you with the graph of Quadratic Function and its
parts.
Parabola - the graph of Quadratic Function
- If 𝒂 > 𝟎 the parabola opens upward and has a minimum point
- If 𝒂 < 𝟎 the parabola opens downward and has a maximum point
Vertex – the turning point of the parabola
Axis of symmetry- is a line that divides the parabola into two parts such that one-half
of the graph is a reflection of the other half.
The Domain of a Quadratic Function is the set of all real numbers.
The Range depends on whether the parabola opens upward or downward.
-If it opens upward the range is the set {𝒚: 𝒚 ≥ 𝒌}
-If it opens downward the range is the set {𝒚: 𝒚 ≤ 𝒌}
In this lesson, we are going to analyze the effect on the graph of a quadratic function
when there is a change in the values of 𝑎, ℎ 𝑎𝑛𝑑 𝑘 in the equation 𝑦 = 𝑎(𝑥 − ℎ)2 + 𝑘.
The vertex of the parabola is represented by (ℎ, 𝑘). The value of 𝑘 is the vertical (𝑦)
location of the vertex and ℎ the horizontal (𝑥 − 𝑎𝑥𝑖𝑠) value. In short, ℎ represent horizontal
shift (how far left or right the graph has shifted from 𝑥 = 0) and 𝑘 represents a vertical shift(
how far up or down the graph has shifted from 𝑦 = 0. ℎ and 𝑘 determines the location of the
curve but not its shape.

Study the graph below


A

A. 𝒚 = 𝟐𝒙𝟐
B. 𝒚 = 𝒙𝟐 B
C. 𝒚 = −𝒙𝟐
D. 𝒚 = −𝟐𝒙𝟐
C

a. Analyze the graphs.


b. What do you notice about the shape of the graph of the quadratic function 𝑦 = 𝑎𝑥 2 ?

94
Note: Practice Personal Hygiene protocols at all times.
c. What happens to the graph as the value of 𝑎 becomes larger/smaller.

d. How do the value of a affects the opening of the graph?

Note! The value of 𝑎 determines the steepness of the parabola. The larger the |𝑎| is, the
narrower is the graph.
C
A. 𝒚 = 𝒙𝟐
E
B. 𝒚 = (𝒙 − 𝟐)𝟐
A
C. 𝒚 = (𝒙 + 𝟐)𝟐 D
D. 𝒚 = (𝒙 − 𝟏)𝟐
B
E. 𝒚 = (𝒙 + 𝟏)𝟐

a. Analyze the graphs


b. What do you notice about the shape of the graph of the quadratic function whose
equation are of the form 𝑦 = (𝑥 − ℎ)2 ?

c. How do you compare the graph of 𝑦 = (𝑥 − ℎ) to 𝑦 = 𝑎𝑥 2 ?

d. Discuss your ideas and observations.

Note! To graph 𝑦 = 𝑎(𝑥 − ℎ)2 , slide the graph of 𝑦 = 𝑎𝑥 2 horizontally 𝒉 units. If ℎ >
0, slide it to the right, if ℎ < 0, slide it to the left. The graph has vertex (ℎ, 0) and its axis is the
line 𝑥 = ℎ.

E
𝟐 B
A. 𝒚=𝒙
B. 𝒚 = 𝒙𝟐 + 𝟐 A
C. 𝒚 = 𝒙𝟐 − 𝟐 C
D. 𝒚 = 𝒙𝟐 − 𝟒 D
E. 𝒚 = 𝒙𝟐 + 𝟒

a. Analyze the graphs


b. What do you notice about the shape of the graph of the quadratic function whose
equation are of the form 𝑦 = 𝑥 2 + 𝑘?
95
Note: Practice Personal Hygiene protocols at all times.
c. How would you compare the graph of 𝑦 = 𝑥 2 + 𝑘 𝑎𝑛𝑑 𝑡ℎ𝑎𝑡 𝑜𝑓 to 𝑦 = 𝑥 2 when
the vertex is above the origin? Below the origin?

d. What conclusion can you give based on your observations.

Note! To the graph of 𝑦 = 𝑎𝑥 2 + 𝑘, slide the graph of 𝑦 = 𝑎𝑥 2 vertically 𝒌 units. If 𝑘 > 0,


slide it upward; if 𝑘 < 0, slide it downward. The graph has vertex (0, 𝑘) and its axis of
symmetry is the line 𝑥 = 0 (𝑦 − 𝑎𝑥𝑖𝑠).

A. 𝒚 = (𝒙 − 𝟐)𝟐 + 𝟒 E
B. 𝒚 = (𝒙 + 𝟑)𝟐 − 𝟒 B
C. 𝒚 = (𝒙 − 𝟏)𝟐 − 𝟑
D
D. 𝒚 = (𝒙 + 𝟒)𝟐 + 𝟓 A
E. 𝒚 = (𝒙 + 𝟐)𝟐 − 𝟐
C

a. Analyze the graphs


b. What is the effect of the variables ℎ and 𝑘 on the graph of 𝑦 = (𝑥 − ℎ)2 + 𝑘 as
compared to the graph of 𝑦 = 𝑥 2 ?

c. Make your generalization on the graph of 𝑦 = (𝑥 − ℎ)2 + 𝑘.

Note! The graph 𝑦 = (𝑥 − ℎ)2 + 𝑘, slide the graph of 𝑦 = 𝑎𝑥 2 horizontally 𝒉 units and
vertically 𝒌 units. The graph has a vertex (ℎ, 𝑘) and its axis of symmetry is the line 𝑥 = ℎ.

Learning Competency: Analyzes the effects of changing the values of 𝑎, ℎ 𝑎𝑛𝑑 𝑘 in the
equation 𝑦 = 𝑎(𝑥 − ℎ)2 + 𝑘 of a quadratic function on its graph (M9AL-Ii-2)

Directions: Read, study carefully and understand the following activities. Answer with
honesty.

96
Note: Practice Personal Hygiene protocols at all times.
Activity 1
Directions: Graph the following quadratic functions and describe their graphs in terms of
the following: Vertex, Opening and the movement of the vertex.
Quadratic Graph Vertex Opening of Movement of
Functions (ℎ, 𝑘) the graph the vertex
A. 𝑦 = 3𝑥 2

B. 𝑦 = −3𝑥 2

A. 𝑦 = (𝑥 + 4)2

B. 𝑦 = (𝑥 − 4)2

Quadratic Graph Vertex Opening of Movement of


Functions (ℎ, 𝑘) the graph the vertex
A. 𝑦 = 𝑥 2 + 1

B. 𝑦 = 𝑥 2 − 1

A. 𝑦 = (𝑥 − 2)2 + 1

B. 𝑦 = (𝑥 + 2)2 + 1
97
Note: Practice Personal Hygiene protocols at all times.
C. 𝑦 = (𝑥 + 2)2 − 1

D. 𝑦 = (𝑥 − 2)2 − 1

Rubrics for Scoring

CRITERIA DISTINGUISHED PROFICIENT APPRENTICE NOVICE RATING


4 3 2 1

Understands the Identifies special Understands the Understands Doesn’t


Problem factors that problem enough to solve understand
influences the part of the problem enough to get
approach before or to get part of the started or make
starting the problem solution progress
Apply Explains why Applies completely Applies some Applies
Appropriate procedures are appropriate appropriate inappropriate
Procedures appropriate for the procedures procedures procedures
problem
Answers the Correct solution of Correct solutions, Clear with No answer or
Problem the problem and clear, concise and appropriate math wrong answer,
made a general rule uses appropriate language, concise cannot explain
about the solution math language the output
and uses
appropriate math
language
Overall Rating

REFLECTION

In this lesson I learned that………………..

REFERENCE
Mathematics 9 Learners Material page 143-145

98
Note: Practice Personal Hygiene protocols at all times.
ANSWER KEY
Activity 1

Quadratic Graph Vertex Opening of Movement of


Functions (𝒉, 𝒌) the graph the vertex

A. 𝒚 = 𝟑𝒙𝟐 (0,0) Upward


A No movement

(0,0) Downward No movement


B. 𝒚 = −𝟑𝒙𝟐
B

A. 𝒚 = (𝒙 + 𝟒)𝟐 (−4,0) Upward Moves 4 units


to the left
A

B
B. 𝒚 = (𝒙 − 𝟒 )𝟐
(4,0) Upward Moves 4 units
to the right

Quadratic Graph Vertex Opening of Movement of


Functions (𝒉, 𝒌) the graph the vertex

A. 𝒚 = 𝒙𝟐 + 𝟏
(0,1) Upward Moves 1 unit
B
upward

(0, −1) Upward Moves 1 unit


B. 𝒚 = 𝒙𝟐 − 𝟏 downward

99
Note: Practice Personal Hygiene protocols at all times.
(2,1) Upward Moves 2 units
A. 𝒚 = (𝒙 − 𝟐)𝟐 + 𝟏 to the right
and 1 unit
upward

(−2,1) Moves 2 units


Upward
B. 𝒚 = (𝒙 + 𝟐 )𝟐 +𝟏 to the left and
one unit
upward

Moves 2 units
(−2, −1) Upward to the left and
1 unit
C. 𝒚 = (𝒙 + 𝟐)𝟐 − 𝟏
downward
A
B
C D
Moves 2 units
(2, −1) Upward to the right
and I unit
D. 𝒚 = (𝒙 − 𝟐)𝟐 − 𝟏 downward

Prepared by:
JOVELYN A. DAQUIOAG
Writer

100
Note: Practice Personal Hygiene protocols at all times.
MATHEMATICS GRADE 9
Name: Grade Level:
Date: Score:

LEARNING ACTIVITY SHEET


Finding the Equation of a Quadratic Function

Background Information for Learners

A. Determines the Equation of Quadratic Function Given a Table of Values


In the previous activities you have learned how to transform Quadratic Function in
general form 𝑦 = 𝑎𝑥 2 + 𝑏𝑥 + 𝑐 into the standard form or vertex form 𝑦 = 𝑎(𝑥 − ℎ)2 + 𝑘 and
vice-versa and familiarized you with its parts. This lesson will teach you on how to find the
equation of a quadratic function given: (a) a table of values; (b) graph; (c) zeros.
You have learned that two points determine a line. This means that if you are given any
two points in the plane, then there is one and only one line that contains both points. A similar
statement can be made about points and quadratic function. A quadratic function, of the
form 𝑦 = 𝑎𝑥 2 + 𝑏𝑥 + 𝑐, is determined by three points. A point is named by its ordered pair of
the form (𝑥, 𝑦) where the first number corresponds to the 𝑥 − 𝑐𝑜𝑜𝑟𝑑𝑖𝑛𝑎𝑡𝑒 and the second to
the 𝑦 − 𝑐𝑜𝑜𝑟𝑑𝑖𝑛𝑎𝑡𝑒. Given three points on the graph of quadratic function, we can work out
the function by finding the values of a, b and c algebraically. This will require solving a system
of three equations in three unknowns.

Note: y = f(x), therefore, 𝑦 = 𝑎𝑥 2 + 𝑏𝑥 + 𝑐 can also be 𝑓 (𝑥 ) = 𝑎𝑥 2 + 𝑏𝑥 + 𝑐

Study the following steps below


STEPS EXAMPLE 1

1. Substitute 3 ordered pairs (𝑥, 𝑦) in 𝑥 −3 −2 −1 0 1


𝑦 = 𝑎𝑥 2 + 𝑏𝑥 + 𝑐 𝑦 4 1 0 1 4
Note! These are the ordered pairs that you
can choose from
(−3,2) (−2,1) (−1,0) (0,1) (1,4)

(−1, 0)
𝑦 = 𝑎𝑥 2 + 𝑏𝑥 + 𝑐
𝑥 = −1 𝑎𝑛𝑑 𝑦 = 0 0 = 𝑎(−1)2 + 𝑏(−1) + 𝑐
𝟎=𝒂−𝒃+𝒄
(0 , 1)
𝑦 = 𝑎𝑥 2 + 𝑏𝑥 + 𝑐
Substitute the values of x and y
1 = 𝑎(0)2 + 𝑏(0) + 𝑐
101
Note: Practice Personal Hygiene protocols at all times.
𝟏=𝒄
(1 , 4)
𝑦 = 𝑎𝑥 2 + 𝑏𝑥 + 𝑐
4 = 𝑎(1)2 + 𝑏(1) + 𝑐
𝟒= 𝒂+𝒃+𝒄
2. Write the 3 equations you came up Eq. 1 𝟎= 𝒂−𝒃+𝒄
with Eq. 2 𝟏=𝒄
Eq. 3 𝟒 =𝒂+𝒃+𝒄
3. Solve for the values of a, b and c The value of c is already solved so let’s find the value
of a and b

Equation 1 and Equation 3


Notice that equation 1 and 3
constitute a linear equation in two
𝟎 =𝒂−𝒃+𝒄
variables so we can use its concept 𝟒= 𝒂+𝒃+𝒄
to eliminate variable/variables
𝟒 = 𝟐𝒂 + 𝟐𝒄
Eq. 4
Substitute the value of c to Equation 4 and solve for a
𝟏 = 𝒄 𝒐𝒓 𝒄 = 𝟏
Equation 4 𝟒 = 𝟐𝒂 + 𝟐𝒄
𝟒 = 𝟐𝒂 + 𝟐(𝟏)
𝟒 = 𝟐𝒂 + 𝟐
Solve for a
Use Addition Property of Equality 4 + (−2) = 2𝑎 + 2 + (−2)
(APE)
2 = 2𝑎
𝒂=𝟏
To solve for b, substitute the values of a and c to either
Equation. 1 or 3

Eq. 3 𝟒= 𝒂+𝒃+𝒄
𝟒 =𝟏+𝒃+𝟏
𝟒=𝒃+𝟐
𝟒 + (−𝟐) = 𝒃 + 𝟐 + (−𝟐)
𝟐=𝒃
𝒂 = 𝟏, 𝒃 = 𝟐, 𝒄=𝟏
4. Write the equation of the quadratic
function 𝑦 = 𝑎𝑥 2 + 𝑏𝑥 + 𝑐 𝒂 = 𝟏, 𝒃 = 𝟐, 𝒄=𝟏
2
𝑦 = 𝑎𝑥 + 𝑏𝑥 + 𝑐
𝑦 = (1)𝑥 2 + (2)𝑥 + (1)
Using the general form of
quadratic function substitute 𝒚 = 𝒙𝟐 + 𝟐𝒙 + 𝟏
the values of a, b and c
Thus ,𝑎 = 1, 𝑏 = 2 𝑎𝑛𝑑 𝑐 = 1 the quadratic function
is 𝒇(𝒙) = 𝒙𝟐 + 𝟐𝒙 + 𝟏

102
Note: Practice Personal Hygiene protocols at all times.
STEPS EXAMPLE 2

1. Substitute 3 ordered pairs (𝑥, 𝑦) in 𝑥 −2 −1 0 2 3


𝑦 = 𝑎𝑥 2 + 𝑏𝑥 + 𝑐 𝑦 0 4 6 4 0
Note! These are the ordered pairs that you
can choose from
(−2,0) (−1,4) (0,6) (2,4) (3,0)

(−2, 0)
𝑦 = 𝑎𝑥 2 + 𝑏𝑥 + 𝑐
Substitute the values of x and y 0 = 𝑎(−2)2 + 𝑏(−2) + 𝑐
𝟎 = 𝟒𝒂 − 𝟐𝒃 + 𝒄
(0 , 6)
𝑦 = 𝑎𝑥 2 + 𝑏𝑥 + 𝑐
6 = 𝑎(0)2 + 𝑏(0) + 𝑐
𝟔=𝒄
𝑥 = 2 𝑎𝑛𝑑 𝑦 = 4 (2 , 4)

𝑦 = 𝑎𝑥 2 + 𝑏𝑥 + 𝑐
4 = 𝑎(2)2 + 𝑏(2) + 𝑐
𝟒 = 𝟒𝒂 + 𝟐𝒃 + 𝒄
2. Write the 3 equations you came up Eq. 1 𝟎 = 𝟒𝒂 − 𝟐𝒃 + 𝒄
with Eq. 2 𝟔=𝒄
Eq. 3 𝟒 = 𝟒𝒂 + 𝟐𝒃 + 𝒄
3. Solve for the values of a, b and c The value of c is already solved so let’s find the value
of a and b
Notice that equation 1 and 3
constitute a linear equation in two Equation 1 and Equation 3
variables so we can use its concept 𝟎 = 𝟒𝒂 − 𝟐𝒃 + 𝒄
to eliminate variable/variables 𝟒 = 𝟒𝒂 + 𝟐𝒃 + 𝒄
Eq. 4 𝟒 = 𝟖𝒂 + 𝟐𝒄

Substitute the value of c to Equation 4 and solve for a


𝟔 = 𝒄 𝒐𝒓 𝒄 = 𝟔
Equation 4 𝟒 = 𝟖𝒂 + 𝟐𝒄
𝟒 = 𝟖𝒂 + 𝟐(𝟔)
Use Addition Property of Equality 𝟒 = 𝟖𝒂 + 𝟏𝟐
(APE) Solve for a
4 + (−12) = 8𝑎 + 12 + (−12)
−8 = 8𝑎
𝒂 = −𝟏

103
Note: Practice Personal Hygiene protocols at all times.
To solve for b, substitute the values of a and c to either
Equation. 1 or 3

Eq. 1 𝟎 = 𝟒𝒂 − 𝟐𝒃 + 𝒄
𝟎 = 𝟒(−𝟏) − 𝟐𝒃 + 𝟔
𝟎 = −𝟒 − 𝟐𝒃 + 𝟔
𝟎 = −𝟐𝒃 + 𝟐
Use Addition Property of Equality
(APE) 𝟎 + −𝟐) = −𝟐𝒃 + 𝟐 + (−𝟐)
(
−𝟐 = −𝟐𝒃
𝟏=𝒃
𝒂 = −𝟏, 𝒃 = 𝟏, 𝒄=𝟔
4. Write the equation of the quadratic
function 𝑦 = 𝑎𝑥 2 + 𝑏𝑥 + 𝑐 𝒂 = −𝟏 𝒃=𝟏 𝒄=𝟔
2
𝑦 = 𝑎𝑥 + 𝑏𝑥 + 𝑐
𝑦 = (−1)𝑥 2 + (1)𝑥 + (6)
Using the general form of
quadratic function substitute 𝒚 = −𝒙𝟐 + 𝒙 + 𝟔
the values of a, b and c
Thus ,𝑎 = −1, 𝑏 = 1 𝑎𝑛𝑑 𝑐 = 6 the quadratic
function is 𝒇(𝒙) = −𝒙𝟐 + 𝒙 + 𝟔

B. Determines the Equation of Quadratic Function Given a Graph


Graph of Quadratic Function of the form 𝑦 = 𝑎𝑥 2 + 𝑏𝑥 + 𝑐 is a curved called parabola.
It also has a turning point called vertex. The vertex of the parabola is the point where the
parabola crosses its axis of symmetry. If a is positive (𝑎 > 0)the parabola opens upward
and has a minimum point, if it is negative (𝑎 < 0)it opens downward and has a maximum
point. If the Quadratic Function is in vertex form 𝑦 = 𝑎(𝑥 − ℎ)2 + 𝑘, the vertex is the point
(ℎ, 𝑘 ) where 𝑥 = 𝑘 is the axis of symmetry and 𝑘 is the minimum or maximum value of
the function.

Study the table below

Axis of
Function Vertex Direction of Opening
Symmetry

a. 𝑦 = 𝑎𝑥 2 (0,0) 𝑥=0 Upward

b. 𝑦 = −𝑎𝑥 2 (0,0) 𝑥=0 Downward

c. 𝑦 = 𝑎(𝑥 − ℎ)2 (ℎ, 0) 𝑥=ℎ Upward

d. 𝑦 = −𝑎(𝑥 − ℎ)2 (ℎ, 0) 𝑥=ℎ Downward

e. 𝑦 = 𝑎𝑥 2 + 𝑘 (0, 𝑘 ) 𝑥=0 Upward

f. 𝑦 = −𝑎𝑥 2 + 𝑘 (0, 𝑘 ) 𝑥=0 Downward

104
Note: Practice Personal Hygiene protocols at all times.
g. 𝑦 = 𝑎(𝑥 − ℎ)2 + 𝑘 (ℎ, 𝑘 ) 𝑥=ℎ Upward

h. 𝑦 = −𝑎(𝑥 − ℎ)2 + 𝑘 (ℎ, 𝑘 ) 𝑥=ℎ Downward


Direction Contains
Vertex Equation of the Parabola
of Opening the point
(0,0) Upward (1,2) Solve for a first.
(1,2) -by replacing (𝑥, 𝑦) with
ℎ=0 - the value 𝑥=1 𝒙 = 𝟏 𝑎𝑛𝑑 𝒚 = 𝟐 and (ℎ, 𝑘 )
of a is with 𝒉 = 𝟎 𝑎𝑛𝑑 𝒌 = 𝟎
𝑘=0 𝑦=2 respectively, we have
positive
(𝑎 > 0)
𝑦 = 𝑎 (𝑥 − ℎ )2 + 𝑘
2 = 𝑎 (1 − 0)2 + 0
2 = 𝑎 (1 )2 + 0
2 =𝑎+0
2=𝑎
𝒂=𝟐

Get the equation of a quadratic


function in the form
𝑦 = 𝑎 (𝑥 − ℎ )2 + 𝑘 by
substituting the obtained values
of 𝑎 and the coordinates of the
vertex in (ℎ, 𝑘) respectively.

𝑦 = 2(𝑥 − 0)2 + 0
𝑦 = 2(𝑥 )2 + 0
𝑦 = 2𝑥 2

Thus the quadratic equation of


the graph on the left is
𝒇(𝒙) = 𝟐𝒙𝟐

Direction Contains
Vertex Equation of the Parabola
of Opening the point
(0,0) (1, −2) Solve for a first.
Downward -by replacing (𝑥, 𝑦) with
ℎ=0 𝑥=1 𝒙 = 𝟏 𝑎𝑛𝑑 𝒚 = −𝟐 and (ℎ, 𝑘 )
- the value with 𝒉 = 𝟎 𝑎𝑛𝑑 𝒌 = 𝟎
𝑘=0 𝑦 = −2 respectively, we have
of a is
negative
𝑦 = 𝑎 (𝑥 − ℎ )2 + 𝑘
(𝑎 < 0) −2 = 𝑎(1 − 0)2 + 0
(1, −2) −2 = 𝑎(1)2 + 0
−2 = 𝑎 + 0
−2 = 𝑎
𝒂 = −𝟐
Get the equation of a quadratic
function in the form
𝑦 = 𝑎 (𝑥 − ℎ )2 + 𝑘 by
substituting the obtained values
of 𝑎 and the coordinates of the
vertex in (ℎ, 𝑘) respectively.
𝑦 = −2(𝑥 − 0)2 + 0
𝑦 = −2(𝑥 )2 + 0
𝑦 = −2𝑥 2
Thus the quadratic equation105 of
Note: Practice Personal Hygiene protocols at all times. the graph on the left is
𝒇(𝒙) = −𝟐𝒙𝟐
Direction Contains
Vertex Equation of the Parabola
of Opening the point
(2,0) Upward (3,2) Solve for a first.
(3,2) -by replacing (𝑥, 𝑦) with
ℎ=2 - the value 𝑥=3 𝒙 = 𝟑 𝑎𝑛𝑑 𝒚 = 𝟐 and (ℎ, 𝑘 )
of a is with 𝒉 = 𝟐 𝑎𝑛𝑑 𝒌 = 𝟎
𝑘=0 𝑦=2 respectively, we have
positive
(𝑎 > 0)
𝑦 = 𝑎 (𝑥 − ℎ )2 + 𝑘
2 = 𝑎 (3 − 2 )2 + 0
(2,0) 2 = 𝑎 (1 ) 2 + 0
2 =𝑎+0
2=𝑎
𝒂=𝟐

Get the equation of a quadratic


function in the form
𝑦 = 𝑎 (𝑥 − ℎ )2 + 𝑘 by
substituting the obtained values
of 𝑎 and the coordinates of the
vertex in (ℎ, 𝑘) respectively.

𝑦 = 2 (𝑥 − 2 )2 + 0
𝑦 = 2 (𝑥 − 2 )2
Thus the quadratic equation of
the graph on the left is
𝒇(𝒙) = 𝟐(𝒙 − 𝟐)𝟐

Direction Contains
(2,0) Vertex Equation of the Parabola
of Opening the point
(2,0) (3, −2) Solve for a first.
Downward -by replacing (𝑥, 𝑦) with
ℎ=2 𝑥=3 𝒙 = 𝟑 𝑎𝑛𝑑 𝒚 = −𝟐
- the value and (ℎ, 𝑘) with 𝒉 =
𝑘=0 𝑦 = −2 𝟐 𝑎𝑛𝑑 𝒌 = 𝟎 respectively, we
of a is
(3, −2) have
negative
(𝑎 < 0) 𝑦 = 𝑎 (𝑥 − ℎ )2 + 𝑘
−2 = 𝑎(3 − 2)2 + 0
−2 = 𝑎(1)2 + 0
−2 = 𝑎 + 0
−2 = 𝑎
𝒂 = −𝟐
Get the equation of a quadratic
function in the form
𝑦 = 𝑎 (𝑥 − ℎ )2 + 𝑘 by
substituting the obtained values
of 𝑎 and the coordinates of the
vertex in (ℎ, 𝑘) respectively.

𝑦 = −2(𝑥 − 2)2 + 0 106


Note: Practice Personal Hygiene protocols at all times. (
𝑦 = −2 𝑥 − 2 ) 2

Thus the quadratic equation of


the graph on the left is
𝒇(𝒙) = −𝟐(𝒙 − 𝟐)𝟐
Direction Contains
Vertex Equation of the Parabola
of Opening the point
(0,2) Upward (1,4) Solve for a first.
(1,4) -by replacing (𝑥, 𝑦) with
ℎ=0 - the value 𝑥=1 𝒙 = 𝟏 𝑎𝑛𝑑 𝒚 = 𝟒 and (ℎ, 𝑘 )
of a is with 𝒉 = 𝟎 𝑎𝑛𝑑 𝒌 = 𝟐
𝑘=2 𝑦=4 respectively, we have
positive
(𝑎 > 0)
4 = 𝑎 (1 − 0)2 + 2
4 = 𝑎 (1 )2 + 2
4 = 𝑎(1) + 2
4 =𝑎+2
4 + (−2) = 𝑎 + 2 + (−2)
2=𝑎

𝒂=𝟐

Get the equation of a quadratic


function in the form
𝑦 = 𝑎 (𝑥 − ℎ )2 + 𝑘 by
substituting the obtained values
of 𝑎 and the coordinates of the
vertex in (ℎ, 𝑘) respectively.

𝑦 = 2(𝑥 − 0)2 + 2
𝑦 = 2(𝑥 )2 + 2

Thus the quadratic equation of


the graph on the left is
𝒇(𝒙) = 𝟐𝒙𝟐 +2

Direction Contains
Vertex Equation of the Parabola
of Opening the point
(0,2) (1,0) Solve for a first.
Downward -by replacing (𝑥, 𝑦) with
ℎ=0 𝑥=1 𝒙 = 𝟏 𝑎𝑛𝑑 𝒚 = 𝟎 and (ℎ, 𝑘)
- the value with 𝒉 = 𝟎 𝑎𝑛𝑑 𝒌 = 𝟐
𝑘=2 𝑦=0 respectively, we have
of a is
negative 𝑦 = 𝑎 ( 𝑥 − ℎ )2 + 𝑘
0 = 𝑎 (1 − 0)2 + 2
(𝑎 < 0) 0 = 𝑎 (1 )2 + 2
0 = 𝑎(1) + 2
0 =𝑎+2
0 + (−2) = 𝑎 + 2 + (−2)
−2 = 𝑎
𝒂 = −𝟐
Get the equation of a quadratic
function in the form
𝑦 = 𝑎 (𝑥 − ℎ )2 + 𝑘 by
substituting the obtained values
of 𝑎 and the coordinates of the
vertex in (ℎ, 𝑘) respectively.
𝑦 = −2(𝑥 − 0)2 + 2107
Note: Practice Personal Hygiene protocols at all times. Thus the quadratic equation of
the graph on the left is
𝒇(𝒙) = −𝟐𝒙𝟐 + 𝟐
Direction Contains
Vertex Equation of the Parabola
of Opening the point
(1,1) Upward (0,3) Solve for a first.
-by replacing (𝑥, 𝑦) with
ℎ=1 - the value 𝑥=0 𝒙 = 𝟎 𝑎𝑛𝑑 𝒚 = 𝟑 and (ℎ, 𝑘)
of a is with 𝒉 = 𝟏 𝑎𝑛𝑑 𝒌 = 𝟏
𝑘=1 𝑦=3 respectively, we have
positive
(𝑎 > 0) 𝑦 = 𝑎 (𝑥 − ℎ )2 + 𝑘
3 = 𝑎 (0 − 1)2 + 1
3 = 𝑎(−1)2 + 1
3 = 𝑎(1) + 1
3 =𝑎+1
3 + (−1) = 𝑎 + 1 + (−1)
2=𝑎
𝒂=𝟐
Get the equation of a quadratic
function in the form
𝑦 = 𝑎 (𝑥 − ℎ )2 + 𝑘 by
substituting the obtained values
of 𝑎 and the coordinates of the
vertex in (ℎ, 𝑘) respectively.
𝑦 = 2 (𝑥 − 1)2 + 1
Thus the quadratic equation of
the graph on the left is
𝒇(𝒙) = 𝟐(𝒙 − 𝟐)𝟐 + 𝟏

Direction Contains
Vertex Equation of the Parabola
of Opening the point
(1,1) (0, −1) Solve for a first.
Downward -by replacing (𝑥, 𝑦) with
ℎ=1 𝑥=0 𝒙 = 𝟎 𝑎𝑛𝑑 𝒚 = −𝟏 and (ℎ, 𝑘 )
- the value with 𝒉 = 𝟏 𝑎𝑛𝑑 𝒌 = 𝟏
𝑘=1 𝑦 = −1 respectively, we have
of a is
negative 𝑦 = 𝑎 (𝑥 − ℎ )2 + 𝑘
−1 = 𝑎(0 − 1)2 + 1
(𝑎 < 0) −1 = 𝑎(−1)2 + 1
−1 = 𝑎(1) + 1
−1 = 𝑎 + 1
−1 + (−1) = 𝑎 + 1 + (−1)
−2 = 𝑎
𝒂 = −𝟐
Get the equation of a quadratic
function in the form
𝑦 = 𝑎 (𝑥 − ℎ )2 + 𝑘 by
substituting the obtained values
of 𝑎 and the coordinates of the
vertex in (ℎ, 𝑘) respectively.
108
𝑦 = −2(𝑥 − 1)2 + 1
Note: Practice Personal Hygiene protocols at all times. Thus the quadratic equation of
the graph on the left is
𝒇(𝒙) = −𝟐(𝒙 − 𝟐)𝟐 + 𝟏
C. Determines the Equation of Quadratic Function Given the Zeros

A root or solution of a quadratic function is the value of the variable that satisfies the
equation. It is also called the zeros of the function. The zeros of a function are points on
the graph of the function where it intersects the x-axis. One way to derive the equation of
a quadratic function is when the zeros are given.

Study the mathematical concept below to have a better understanding on how to get the
equation of the quadratic function given its zeros
Problem 1: If 2 and 3 are zeros of a quadratic function, find the equation of the quadratic
function.
The following are two different ways of determining equation of the quadratic function given
its zeros.

Using Factored Form


If 𝑟1 𝑎𝑛𝑑 𝑟2 are the zeros of a quadratic function then 𝑓 (𝑥 ) = 𝑎(𝑥 − 𝑟1 )(𝑥 − 𝑟2 ) where a is a
non zero constant that can be determined from the other point on the graph.
𝑟1 = 2

𝑟2 = 3

𝑓 (𝑥 ) = 𝑎(𝑥 − 𝑟1 )(𝑥 − 𝑟2 )
Note!
Substitute the given roots then
multiply the binomial or apply 𝑓(𝑥) = 𝑎(𝑥 − 𝑟1 )(𝑥 − 𝑟2 ),
𝑓 (𝑥 ) = 𝑎(𝑥 − 2)(𝑥 − 3) where a is any nonzero constant
FOIL method
𝑓 (𝑥 ) = 𝑎(𝑥 2 − 5𝑥 + 6)

𝒇(𝒙) = 𝒙𝟐 − 𝟓𝒙 + 𝟔

Using the sum and product of roots

You can use the sum and product of the zeros to find the equation of the quadratic function.
𝑓(𝑥 ) = 𝑥 2 − (𝑠𝑢𝑚 𝑜𝑓 𝑟𝑜𝑜𝑡𝑠)𝑥 + 𝑝𝑟𝑜𝑑𝑢𝑐𝑡 𝑜𝑓 𝑟𝑜𝑜𝑡𝑠
Sum of Roots: 𝑥1 + 𝑥2 = 2 + 3 = 𝟓
Product of Roots: 𝑥1 ∙ 𝑥2 = 2 ∙ 3 = 𝟔
𝑓(𝑥 ) = 𝑥 2 − (𝑠𝑢𝑚 𝑜𝑓 𝑟𝑜𝑜𝑡𝑠)𝑥 + 𝑝𝑟𝑜𝑑𝑢𝑐𝑡 𝑜𝑓 𝑟𝑜𝑜𝑡𝑠
Substitute the computed sum 𝑓 (𝑥 ) = 𝑥 2 − (5)𝑥 + 6
and product of roots 𝒇(𝒙) = 𝒙𝟐 − 𝟓𝒙 + 𝟔

Thus an equation of quadratic function with zeros 2 and 3 is 𝒇(𝒙) = 𝒙𝟐 − 𝟓𝒙 + 𝟔

109
Note: Practice Personal Hygiene protocols at all times.
Problem 2: Find an equation of the quadratic function whose zeros are ± 3√5.
Suggestion: Use sum and product of Roots
Solutions: 𝑓 (𝑥 ) = 𝑥 2 − (𝑠𝑢𝑚 𝑜𝑓 𝑟𝑜𝑜𝑡𝑠)𝑥 + 𝑝𝑟𝑜𝑑𝑢𝑐𝑡 𝑜𝑓 𝑟𝑜𝑜𝑡𝑠
Sum of Roots: 𝑥1 + 𝑥2 = 3√5 + (− 3√5) = 𝟎
Product of Roots: 𝑥1 ∙ 𝑥2 = 3√5 ∙ (−3√5) = −𝟒𝟓
𝑓(𝑥 ) = 𝑥 2 − (𝑠𝑢𝑚 𝑜𝑓 𝑟𝑜𝑜𝑡𝑠)𝑥 + 𝑝𝑟𝑜𝑑𝑢𝑐𝑡 𝑜𝑓 𝑟𝑜𝑜𝑡𝑠
𝑓 (𝑥 ) = 𝑥 2 − (0)𝑥 + (−45)
𝒇(𝒙) = 𝒙𝟐 − 𝟒𝟓
Thus an equation of quadratic function with zeros ± 3√5 is 𝒇(𝒙) = 𝒙𝟐 − 𝟒𝟓
1 3
Problem 3: Find an equation of the quadratic function whose zeros are 𝑎𝑛𝑑 − .
2 2

Suggestion: Use Factored Form


Solutions:
1
𝑟1 =
2
3
𝑟2 = −
2
𝑓(𝑥) = 𝑎(𝑥 − 𝑟1 )(𝑥 − 𝑟2 )
1 3
𝑓(𝑥) = 𝑎 (𝑥 − ) (𝑥 − (− ))
2 2
1 3
𝑓(𝑥) = 𝑎 (𝑥 − ) (𝑥 + )
2 2
𝑓(𝑥) = 𝑎(2𝑥 − 1)(2𝑥 + 3)
𝑓(𝑥) = 𝑎(4𝑥 2 + 6𝑥 − 2𝑥 − 3)
𝑓(𝑥) = 𝑎(4𝑥 2 + 4𝑥 − 3)
𝑓(𝑥) = 4𝑥 2 + 4𝑥 − 3
1 3
Thus an equation of quadratic function with zeros 𝑎𝑛𝑑 − 2 is 𝒇(𝒙) = 𝟒𝒙𝟐 + 𝟒𝒙 − 𝟑
2

Learning Competency: Determines the equation of a quadratic function given: (a) a table of
values (b) graphs (c) zeros

Directions: Read, study carefully and understand the following activities. Answer with
honesty.

110
Note: Practice Personal Hygiene protocols at all times.
Activity 1
Directions: Use the following steps below to determine the equation of a quadratic function
given a table of values
STEPS TASK 1

1. Substitute 3 ordered pairs 𝑥 −3 −2 −1 0 1


(𝑥, 𝑦) in 𝑦 = 𝑎𝑥 2 + 𝑏𝑥 + 𝑐 𝑦 5 2 1 2 5

2. Write the 3 equations you came up


with
3. Solve for the values of a, b and c

4. Write the equation of the quadratic


function 𝑦 = 𝑎𝑥 2 + 𝑏𝑥 + 𝑐

STEPS TASK 2

1. Substitute 3 ordered pairs (𝑥, 𝑦) in 𝑥 −3 −1 0 1 2


𝑦 = 𝑎𝑥 2 + 𝑏𝑥 + 𝑐 𝑦 10 0 2 2 0

111
Note: Practice Personal Hygiene protocols at all times.
2. Write the 3 equations you came up
with

3. Solve for the values of a, b and c

4. Write the equation of the quadratic


function 𝑦 = 𝑎𝑥 2 + 𝑏𝑥 + 𝑐

Activity 2

Directions: Complete the following table.

Graph Direction Contains Equation of the


Vertex
of Opening the point Parabola
Graph 1

(1,3)

112
Note: Practice Personal Hygiene protocols at all times.
Graph 2

(1, −3)

Graph Direction Contains Equation of the


Vertex
of Opening the point Parabola
Graph 3

113
Note: Practice Personal Hygiene protocols at all times.
Graph 4

Graph Direction Contains Equation of the


Vertex
of Opening the point Parabola
Graph 5

(1,4)

114
Note: Practice Personal Hygiene protocols at all times.
Graph 6

(1, −2)

Graph Direction Contains Equation of the


Vertex
of Opening the point Parabola
Graph 7

115
Note: Practice Personal Hygiene protocols at all times.
Graph 8

Activity 3:
Directions: To get the hidden message write the indicated letter of the quadratic function on
the corresponding blank below the box containing the zeros of the function

R 𝑓 (𝑥 ) = 𝑥 2 − 6𝑥 + 8 A 𝑓 (𝑥 ) = 𝑥 2 − 𝑥 − 20
V 𝑓 (𝑥 ) = 9𝑥 2 − 16 E 𝑓 (𝑥 ) = 𝑥 2 + 2𝑥 − 3
I 𝑓 (𝑥 ) = 𝑥 2 − 12 N 𝑓 (𝑥 ) = 𝑥 2 − 4
G 𝑓(𝑥 ) = 6𝑥 2 − 7𝑥 + 2 S 𝑓 (𝑥 ) = 12𝑥 2 − 7𝑥 − 12
L 𝑓 (𝑥 ) = 𝑥 2 − 5𝑥 − 36 T 𝑓 (𝑥 ) = 4𝑥 2 − 9
O 𝑓(𝑥 ) = 𝑥 2 − 45 P 𝑓 (𝑥 ) = 𝑥 2 − 4𝑥 − 21

4 4
{𝟒, 𝟐} {−𝟑, 𝟏} { ,− } {−𝟑, 𝟏} {𝟐, −𝟐}
3 3

3 3 3 4
{−𝟑, 𝟕} {±𝟑√𝟓} { ,− } {− , }
2 2 4 3

1 2
{ , } {𝟐, −𝟐} {±𝟐√𝟑} {𝟐, −𝟐} {𝟒, 𝟐} {𝟓, −𝟒} {−𝟑, 𝟏} {𝟗, −𝟒}
2 3

116
Note: Practice Personal Hygiene protocols at all times.
RUBRICS FOR THE ACTIVITY

CRITERIA DISTINGUISHED PROFICIENT APPRENTICE NOVICE RATING


4 3 2 1

Understands the Identifies special Understands the Understands Doesn’t


Problem factors that problem enough to solve understand
influences the part of the problem enough to get
approach before or to get part of the started or make
starting the problem solution progress
Apply Explains why Applies completely Applies some Applies
Appropriate procedures are appropriate appropriate inappropriate
Procedures appropriate for the procedures procedures procedures
problem
Answers the Correct solution of Correct solutions, Clear with No answer or
Problem the problem and clear, concise and appropriate math wrong answer,
made a general rule uses appropriate language, concise cannot explain
about the solution math language the output
and uses
appropriate math
language
Overall Rating

Reflection

In this lesson I learned that………………..

References
Mathematics 9 Learners Material page 156-173
Exploration Application Workbook page 51-53

117
Note: Practice Personal Hygiene protocols at all times.
ANSWER KEY
Activity 1
Task 1 𝑓(𝑥) = 𝑥 2 + 2𝑥 + 2
Task 2 𝑓(𝑥) = −𝑥 2 + 𝑥 + 2

Activity 2

Vertex Direction of the Contains the Equation of the Parabola


Opening point
Graph 1 (0,0) Upward (1,3) 𝑓(𝑥) = 3𝑥 2
Graph 2 (0,0) Downward (1, −3) 𝑓(𝑥) = −3𝑥 2
Graph 3 (1,0) Upward (0,3) 𝑓(𝑥) = 3(𝑥 − 1)2
Graph 4 (1,0) Downward (0, −3) 𝑓(𝑥) = −3(𝑥 − 1)2
Graph 5 (0,1) Upward (1,4) 𝑓(𝑥) = 3𝑥 2 + 1
Graph 6 (0,1) Downward (1, −2) 𝑓(𝑥) = −3𝑥 2 + 1
Graph 7 (1,1) Upward (0,4) 𝑓(𝑥) = 3(𝑥 − 1)2 + 1
Graph 8 (1,1) Downward (0, −2) 𝑓(𝑥) = −3(𝑥 − 1)2 + 1

Activity 3

Hidden Message: R E V E N
P O T S
G N I N R A E L

(Read each word backward) NEVER STOP LEARNING

Prepared by:
JOVELYN A. DAQUIOAG
Writer

118
Note: Practice Personal Hygiene protocols at all times.
MATHEMATICS GRADE 9
Name of learner: _________________________________ Grade Level: _____________
Section: ________________________________________ Date: _______________

LEARNING ACTIVITY SHEET


Problem Solving Involving Quadratic Functions

Background Information for Learners


This lesson was about solving real-life problems involving quadratic functions. The
lesson provided for you with opportunities to see the real-life applications of quadratic
functions in this two day-lesson.
Quadratic function can be applied in different fields like physics, industry, and business
and in various other mathematical problems. Familiarity with quadratic function, their zeros
and their properties is very important in solving real-life problems.
As with any other algebraic problems, we always represent all unknowns with the use
of variables. We then construct a mathematical model that describes to the problem. We must
always note what each of the variables pertain to, to avoid confusion.
To better understand how the concepts of the quadratic function can be applied to solve
geometry problems, study the illustrative example presented below.

Example 1. What are the dimensions of the largest rectangular field that can be
enclosed by 80 m of fencing wire?
Solution: Let l and w be the length and width of a rectangle. Then, the perimeter P of
a rectangle is P = 2l + 2w. Since P = 80 m, thus,

2l + 2w = 80
l + w = 40 It follows that
l = 40 – w expressing the length as a function of w
l = 40 – w Substituting in the formula for the area A of a rectangle
w A(w) = wl
A(w) = w ( 40 – w )
A(w) = -w2+ 40w
By completing the square, A(w) = - (w - 20 )2 + 400. The vertex of the graph of the
function A(w) is (20, 400). This point indicates the maximum value of 400 for A(w) that occurs
when w = 20. Thus, the maximum area is 400 m 2 when the width is 20. If the width is 20 m,
then the length is (40 – 20) m or 20 m also. The field with maximum area is a square.

The illustrative example below is intended for you to better understand the key ideas
necessary to solve real-life problems involving quadratic function.
Free falling objects can be modeled by a quadratic function h(t) = –4.9t2 + V0t + h0,
where h(t) is the height of an object at t seconds, when it is thrown with an initial velocity of
V0 m/s and an initial height of h0 meters. If units are in feet, then the function is h(t) = –16t2 +
V0t + h0.
119
Note: Practice Personal Hygiene protocols at all times.
Illustrative example:
From a 96-foot building, an object is thrown straight up into the air then follows a
trajectory. The height S(t) of the ball above the building after t seconds is given by the function
S(t) = 80t – 16t2. 1. What maximum height will the object reach? 2. How long will it take the
object to reach the maximum height? 3. Find the time at which the object is on the ground.
Solution 1. The maximum height reached by the object is the ordinate of vertex of the
parabola of the function S(t) = 80t – 16t2. By transforming this equation in completed square
form we have, S(t) = 80t – 16t2
S(t) = – 16t2 + 80t
S(t) = – 16(t2 - 5t)
25
S(t) = – 16(t2 - 5t + ) + 100
4
5 2
S(t) = – 16(t - ) + 100
2
5
The vertex is (2 , 100). Thus the maximum height reached by the object is 100 ft from
the top of the building. This is 196 ft. from the ground.

2. The time for an object to reach the maximum height is the abscissa of the vertex of
the parabola or the value of h.
S(t) = 80t – 16t2
5 2
S(t) = – 16(t - ) + 100
2
5
Since the value of h is or 2.5, then the object is at its maximum height after 2.5
2
seconds.

3. To find the time it will take the object to hit the ground, let S(t) = -96, since the height
of the building is 96 ft. The problem requires us to solve for t.
h(t) = 80t – 16t2
-96= 80t – 16t2
16t2 -80t -96 = 0
t2 – 5t - 6 = 0
( t -6) ( t + 1) = 0
t = 6 or t = -1
Thus, it will take 6 seconds before the object hits the ground.

A quadratic function can be applied also in business/industry to determine the


maximum profit, the break-even situation and the like. Suppose x denotes the number of units
a company plans to produce or sell. The revenue function R(x) is defined as R(x)= (price per
unit) x (number of units produced or sold). Study the example below.
Illustrative example
A garments store sells about 40 t-shits per week at a price of Php100 each. For each
P10 decrease in price, the sales lady found out that 5 more t-shits per week were sold.
a. Write a quadratic function in standard form that models the revenue from t-shirt
sales.
b. What price produces the maximum revenue?

120
Note: Practice Personal Hygiene protocols at all times.
Solution: You know that Revenue R(x) = (price per unit) x (number of units produced
or sold). Therefore, Revenue R(x) = (Number of t-shirts sold) (Price per t-shirt)
Revenue R(x) = (40 + 5x) (100-10x)
R(x) = -50x2 + 100x +4000
If we transform the function into the form y = a(x- h)2 + k
R(x) = -50(x -1)2 + 4050
The vertex is (1, 4050). Thus, the maximum revenue is Php 4050

The price of the t-shirt to produce maximum revenue can be determined by


P(x) = 100 – 10x
P(x) = 100 – 10 (1) = 90
Thus, Php 90 is the price of the t-shirt that produces maximum revenue.

Learning Competency with Code:


Solves problems involving quadratic functions (M9AL-Ii-j-2)

Directions: Read, study carefully and understand the following activities. Answer with
honesty.

Activity 1: Hit the mark!


Directions: Analyze and solve this problem.

1. A company of cellular phones can sell 200 units per month at P 2 000 each. Then they
found out that they can sell 50 more cell phone units every month for each P 100
decrease in price.
a. How much is the sales amount if cell phone units are priced at P2000 each?
b. How much would be their sales if they sell each cell phone unit at P 1600?
c. Write an equation for the revenue function?
d. What price per cell phone unit gives them the maximum monthly sales?
e. How much is the maximum sale?

2. The ticket to a film showing costs P 20. At this price, the organizer found out that all
the 300 seats are filled. The organizer estimates that if the price is increased, the number
of viewers will fall by 50 for every P 5 increase.
a. What ticket price results in the greatest revenue?
b. At this price, what is the maximum revenue?

Your goal in the next section is to have a deeper understanding on how to solve
problems involving quadratic functions. The activities provided for you in this section will be
of great help to practice the key ideas developed throughout the lesson and to stimulate your
synthesis of the key principles and techniques in solving problems on quadratic functions.

121
Note: Practice Personal Hygiene protocols at all times.
Activity2: Geometry and Number!
Directions: Solve the problems.
1. What are the dimensions of the largest rectangular field that can be enclosed with 60
m of wire?
2. Find the maximum rectangular area that can be enclosed by a fence that is 364 meters
long.
3. Find two numbers whose sum is 36 and whose product is a maximum.
4. The sum of two numbers is 28. Find the two numbers such that the sum of their
squares is a minimum?
5. Marlon wants to fence a rectangular area that has one side bordered by an irrigation.
If he has 80 m of fencing materials, what are the dimensions and the maximum area he
can enclose?

Now let us try your skill in applying your understanding quadratic functions in the field
of physics.
Activity 3: It’s high time!
Directions: Solve the problems. Show your solution.
1. A ball is launched upward at 14 m/s from a platform that is 30 m high.
a. Find the maximum height the ball reaches.
b. How long it will take the ball to reach the maximum height?
c. How long will it take the ball to reach the ground?

2. On top of a hill, a rocket is launched from a distance 80 feet above a lake. The rocket
will fall into the lake after its engine burns out. The rocket’s height h, in feet above the
surface of the lake is given by the equation h = -16t2 + 64t +80, where t is time in
seconds. What is the maximum height reached by the rocket?

3. A ball is launched upward at 48 ft/s from a platform that is 100 ft. high. Find the
maximum height the ball reaches and how long it will take to get there.

And finally, try to apply quadratic functions in business or industry. Common, you can
do it!
Activity 4: Reach the target!
Directions: Solve the problems below. Show your solution.
1. A store sells lecture notes, and the monthly revenue R of this store can be modelled
by the function R(x) = 3000 +500x -100x2, where x is the peso increase over Php 4.
What is the maximum revenue?
2. A convention hall has a seating capacity of 2000. When the ticket price in the concert
is Php160, attendance is 500. For each Php 20 decrease in price, attendance increases by
100.
a. Write the revenue R of the theater as a function of concert ticket price x.
b. What ticket price will yield maximum revenue?
c. What is the maximum revenue?
3. A smart company has 500 customers paying P 600 each month. If each Php 30
decrease in price attracts 120 additional customers, find the approximate price that
yields maximum revenue?
122
Note: Practice Personal Hygiene protocols at all times.
Rubrics for Scoring
Indicators
A. Representation
B. Mathematical Equation
C. Algebraic Solution
D. Correct Answer
5 points – All the indicators are correct
4 points – atleast three of the indicators are correct
3 points - atleast two of the indicators are correct
2 points - atleast one of the indicator is correct
1 point – none of the indicator is correct but attempted to solve/answer
the problem
Reflection
What new insights do you have about the real-life applications of quadratic functions
and how would you connect this to your daily life?
___________________________________________________________________________
___________________________________________________________________________
___________________________________________________________________________
___________________________________________________________________________
References
o Grade 9 Learning Module pp.194-205
o http://www.algebra.com/algebra/homework/quadratic/lessons/Using-
quadratic-fnctions-to-solve-word-problems.lesson

123
Note: Practice Personal Hygiene protocols at all times.
Answer Key
Activity 1: Hit the Mark!
1.
a. Php 400,000.00
b. Php 640,000.00
c. Equation R(x) = (200 + 50x) (2000 – 100x)
d. Php 1,200.00
e. Php 720,000.00
2.
a. Php 25.00
b. Php 6,250.00

Activity 2: Geometry and Number!


1. 15m by 15m
2. 8281 m2
3. 18 and 18
4. 14 and 14
5. 20m by 40m

Activity 3: It’s High Time!


1.
a. 40m
𝟏𝟎
b. 𝟕
c. 4.28 seconds
2. 144ft
3. 136ft – maximum height and 1.5 seconds
Activity 4: Reach the target!
1. Php 3,625
2.
a. R(x) = (500 +100x)(160 – 20x)
b. Php 130
c. Php 84,500
3. Php 363.00

Prepared by:

SHERYL JOY P. GALVEZ


Writer

124
Note: Practice Personal Hygiene protocols at all times.

Das könnte Ihnen auch gefallen